Vous êtes sur la page 1sur 92

PREVENTIVE, FAMILY & COMMUNITY MEDICINE

EPIDEMIOLOGY, HEALTH STATISTICS & RESEARCH

Note: Answers from Numbers 1-96 are all letter


A.
1. The 95 percent confidence interval for a sample mean value
represents:
A. The range in which the true population mean is most likely to
exist.
B. The authors belief that the presented data are true.
C. The range in which 95 percent of the sample values fall.
D. A test for statistical significance of the mean value.
2. OF the following statements, which best describes the specificity of a
clinical test?
A. The proportion of non-diseased individuals who have a negative
test
B. The proportion of diseased individuals who have a positive test
C. The proportion of diseased and non-diseased individuals who are
correctly categorized by the test
D. The proportion of test-negative individuals who are free of
disease
Data from clinical studies are reported frequently in the form of 2 by 2
tables. A 2 by 2 table for a hypothetical test is shown below:
Test Result
Disease Present
Disease Absent
Positive
50
20
Negative
30
50
3. What is the test sensitivity?
A. 63 percent
B. 29 percent

C. 71 percent
D. 38 percent

4. What is the positive predictive value?


A. 71 percent
C. 38 percent
B. 47 percent
D. 33 percent
5. You have decided to conduct a research project in your office. A
number of steps should be taken to assure a successful study. What is
the first step you should complete in planning your project?
A. Formulate the study question.
B. Consult with a statistician.
C. Design a form for collecting data.
D. Determine the necessary sample size.
6. You have conducted a clinical trial in which you measured blood
pressure in the same patients during a control period and after 2
weeks of treatment with an experimental antihypertensive medication.
Systolic blood pressure was the characteristic you measured. You now
want to determine whether there was a significant difference in the
average (mean) systolic blood pressure between the control and
treatment periods. Which of the following is the most appropriate
statistical procedure for this determination?
A. Paired t test
C. two sample t test
B. Chi-square
D. correlation coefficient
1

7. Experimental studies:
A. Are the most effective studies to assess causal relationships.
B. Are synonymous with observational studies.
C. Require that subjects be randomly assigned to exposure and
nonexposure groups at all times.
D. Excludes community trials.
8. Which of the following statements is true regarding prospective cohort
studies?
A. They measure the relative risk of developing a disease for an
individual with an exposure.
B. Individuals are grouped on the basis of their disease status.
C. Subject attrition is not a major potential flaw.
D. They cannot be used to observe multiple exposures and
conditions.
E. They are poor in elucidating temporal relationships between
exposure and disease.
9. Which of the following statements is true regarding cross-sectional
studies?
A. They assess exposure and disease status simultaneously.
B. They can yield information regarding disease incidence.
C. They are more time-consuming to perform than case-control or
cohort studies.
D. They are an effective design to determine casual relationships.
E. They are not an appropriate methodology to draw conclusions
from chart databases or census data.
10.

Which of the following is an example of cohort studies?


A. Men are grouped as to whether or not they have had radioactive
exposure; then their existing medical records are evaluated for a
diagnosis of lung cancer.
B. Women are grouped as to whether or not they have chronic
obstructive lung disease; then they are asked about their
occupational history.
C. Women are given a questionnaire assessing weight and
menstrual symptoms.
D. A group of elderly veterans are followed for adverse reactions to
the flu vaccine.

11.
A family physician believes that sterile butterfly stripping is just
as effective as suturing with regard to the closure of simple
lacerations. To test his theory, he uses sterile butterfly strips to close
the next 25 lacerations that he sees in his office. He reports that 96
percent of the lacerations had excellent results. Which of the following
statements is true regarding this experiment?
A. This is a case-series design.
B. This design is an ineffective one for the initial study of the
phenomenon.
C. It can be assumed that all lacerations were equivalent with
regard to location, extent, and mechanism of injury.
D. The next phase of study should be a prevalence design.
E. Sterile butterfly stripping is at least equivalent to suturing with
regard to its ability to attain adequate wound closure.
12.

In double-blind randomized clinical trials:


2

A. The distribution of characteristics between the exposure and


nonexposure groups is determined by chance.
B. The investigator is aware of who is and who is not getting the
exposure.
C. The subjects are not aware that they are taking part in a
research protocol.
D. The subjects are permitted to choose whether they want to be in
the exposure or nonexposure group.
OCCUPATIONAL HEALTH, INDUSTRIAL MEDICINE & MEDICAL
SOCIOLOGY
13.

The following agent/s causing diffuse mottling of the lungs:


A. Bagasse
C. Silica
B. Candida
D. TB

14. Substitute/s causing neurological problems:


A. Mercury
C. Asbestos
B. Silica
D. Bagasse
15.

Pneumoconiosis is caused by:


A. Asbestos
C. Bagassosis
B. Cotton dust
D. Bagasse

16.
This mineral dust is incurable and may lead to the development
of Tuberculosis:
A. Silica
C. Cryptococcus
B. Histoplasma
D. Aspergillosis
17.
A.
B.
C.
D.

An important component of an occupational history is:


temporal relationship of symptoms with work time
associated symptoms among family members
family illnesses
dietary history

18. The most exposed industry to ergonomic problems:


A. Manufacturing
C. Fishing
B. Mining
D. Agriculture
19. Threshold limit value is the time weighted average concentration
which nearly all workers can be exposed daily without adverse effect
for the following number of work hours/workweek:
A. 40-hour workweek
C. 48-hour workweek
B. 12-hour workday
D. 6 hours work shift
SANITATION, ENVIRONMENTAL MEDICINE & HUMAN ECOLOGY
20.
Considered most dangerous with regards to possible disease
spread:
A. clinical carrier
C. non-immunized
B. clinical stage patient
D. terminally ill patient
21.
This allows organic material to undergo bio and photo
degradation, resulting in simple organic molecules that can actually be
beneficial to the environment:
A. Composting
C. Source reduction

B. Reuse and recycling


22.

D. All of the above

An example of an agent-environment interaction is:


A. Population of a carrier making use of an infected water supply
both for drinking and food preparation
B. Vitamin C tablets placed in colored bottles
C. Fly control program
D. Businesss favorite of eating sushi in Japanese restaurants

23. What treatment is needed if the bacteriological quality of water is


5,00050,000 coliforms/100 ml?
A. Double treatment
C. Disinfection
B. Look for another source
D. Chlorination
24. The most critical step in waste reduction and recycling is:
A. Waste segregation
C. collection of solid waste
B. incineration
D. transfer and transport
25. The main objective of solid waste processing is:
A. to recover whatever may be reusable
B. to provide livelihood for scavengers
C. to discover other sources of income
D. to minimize bulk of waste to be dumped at sea
26. The most important public health issues addressed by correct waste
management:
A. Composting of solid waste from laboratories
B. Control of vectors of infectious disease
C. Eradication of mosquito causing Dengue
DC. Improvement of parks and public places
FAMILY MEDICINE
27. Biopsychosocial Model focuses on:
A. Multifactorial causes of
C. Organ system dysfunction
illness
B. Disease oriented
D. Biological orientation
28.
The most challenging and rewarding stage for the physician in
the Family Illness Trajectory:
A. Major therapeutic efforts
B. Accuracy of diagnosis
C. Early adjustment to possible outcome
D. Adjustment to permanency of outcome
29.
It is process which encompasses screening for abnormalities,
early detection of disorders that can be alleviated, and likewise the
prevention of ill-health:
A. Family Health Care
C. Family Illness Trajectory
B. Family Life Cycle
D. Family Assessment Tools
30.
The nature of illness whose characteristics of experience
provide little time for physical and psychological adjustment for family
members:
A. Acute illness
C. Terminal illness
B. Chronic illness
D. Debilitating illness

31.
The most difficult stage of the Family Illness Trajectory for the
patient:
A. Accuracy of diagnosis
B. Adjustment to permanency of outcome
C. Major therapeutic efforts
D. Early adjustment to possible outcome
32.
This household is in the typical case-economically independent
subsisting in the first instance from the occupational earnings of the
husband-father:
A. Nuclear family
C. Extended family
B. Corporate family
D. Blended family
33.
An acronym that stands for factors affecting health which can be
considered resource and pathology:
A. SCREEM
C. Family APGAR
B. Family Circle
D. Family Genogram
34.
Component of the Family Genogram that projects the dynamism
of family
relationships:
A. Functional chart
C. Family Illness and history
B. Family resources
D. Pedigree or Family Tree
35.
A.
B.
C.
D.

A Genogram using functional symbols:


depicts relationships
focuses on the most ill family member
emphasizes the problem of the patient
therapeutic interventions

36.
The therapeutic Triangle in Medicine refers to which of the
following?
A. Patient/family/health care team
B. Patient/family, health care institution, health care team
C. Patient/family, friends/extended family, health care team
D. Patient, health care team, community
37.
What stage of the Family Life Cycle is characterized by parents
developing adult type relationships between the grown children and
themselves?
A. Family with adolescent
C. Unattached Young Adult
B. Newly married couple
D. Family in later life
38.
What family process involves ordered sequences of interaction
that typify how family functions?
A. Family pattern
C. Triangulation
B. Disengagement
D. Enmeshment
39.
A.
B.
C.
D.
40.

Which of the following is characteristic of the hospice program?


available 24 hours a day
designed for marginalized population
renders only medical care
composed of nurse and physician only
During the interview process, which of the following will most
5

likely put the patient on the defensive, that is, make the patient feel
that his or her problem or behavior must be justified and defended?
A. Beginning the question with
C. Laundry-list questions
Why?
B. Direct question
D. Open-ended questions
41.
This is a component of the family health care plan which involves
health maintenance for each Family member:
A. Preventive
C. Therapeutic
B. Diagnostic
D. Rehabilitative
42. A family meeting should be done in the following situation:
A. 16 y/o old female, first pregnancy
B. 12 y/o female with acne
C. 4 y/o old boy with acute nasopharyngitis
D. 30 y/o old male for annual employment physical
HEALTH ADMINISTRATION & MANAGEMENT, PUBLIC HEALTH
43.
A characteristic of Primary health care that differentiates it from
the traditional health delivery system:
A. Community participation in the stages of planning
B. Assurance of financial backing of the government
C. Coverage of the majority of the population
E. Drug provision for all indigent patients
44.
Fecal occult blood is a screening test for:
A. colorectal CA
C. Inflammatory bowel disease
B. cervical cancer
D. hemorrhoids
45.
Criteria that justifies doing screening test:
A. Can identify patients likely to have the disease at pre-symptomatic
stage
B. Can identify patients who have the disease at symptomatic stage
C. Can identify patients who will not benefit from treatment
D. Can identify patients who will not benefit from rehabilitation
46.
Which of the health maintenance plan is true for adult patients?
A. FBS should be done for obese 40 y/o female with family history of
hypertension
B. Fecal occult blood is recommended for 50 years old and above
C. VDRL/RPR should be done on all males
D. Electrocardiogram should be done on all 50 years old and above
47.
Which of the following statements about chemoprophylaxis is true?
A. Low dose aspirin for men 65 years old and above prevents heart
disease
B. Estrogen with progestin increases the risk of endometrial cancer
C. Estrogen replacement therapy decrease the risk of osteoporosis
when started in women 40 years old and below
D. Chemoprophylaxis is warranted only in younger patients
48. A
A.
B.
C.

health promotion measure:


Utilization of health/medical services
Use of mosquito repellant
Vitamin supplementation to increase body resistance

D. Isolating patients with infectious diseases


49. A rehabilitation measure:
A. Organized feeding program
B. Use of iodized salt because of prevalence of goiter
C. Teaching to deaf mute children sign language or/and lip reading
D. Prescribing eyeglasses to those with initial stage of
nearsightedness (myopia)
50. The process of isolating patients with Flu falls under:
A. Specific protection
C. Disability limitation
B.
Early diagnosis and D. Health promotion
treatment
51. Factors affecting community reaction:
A. Previous exposure
C. Virulence of agent
B. Chance contact
D. Volume of agent
52. An
A.
B.
C.
D.

epidemic occurs due to:


Absence of an immune barrier
Inadequacy of therapeutic measures
Emergence of a new strain of the disease agent
Migration of a population group

53. Considered a missed human reservoir of disease:


A. Undiagnosed and therefore unreported case of a disease
B. Patient wrongly diagnosed because of inadequate laboratory workup
C. Seek medical consultation
D. Correctly diagnosed patient but unreported case
54. Increase in life expectancy is mainly due to:
A. Decrease in mortality in the younger age groups
B. Improvement in health information dissemination
C. Better diagnostic facilities
D. Discovery of effective drugs
55. Preventive measures are most effective when applied to a person:
A. unaffected
C. affected symptomatic
B. affected asymptomatic
D. without complications
56. Milas father and mother both have diabetes. She consulted because
of polyuria. Which determinant of health is explored in the history?
A. Genetic inheritance
C. Social factors
B. Belief systems
D. Nutrition
57. Denotes a non-human carrier of infections organisms that can
transmit disease directly to humans:
A. Vectors
C. Plasmodium Sp.
B. Rodents
D. All of the above
58.
A.
B.
C.
D.

Step in epidemiological method of investigation include:


Discovering historical occurrence of diseases
Formulation of community diagnosis of health and disease
Estimation of morbidity and mortality rates
Making critical appraisal of existing information

59. Which statement is TRUE about the Control of Diarrheal disease


program?
A. Emphasizes home therapy by continued feeding and increase
fluid intake
B. Aims to eradicate food and water borne diseases
C. Targets children under 5 - 7 years old
D. Promotes the use of anti-diarrheal drugs as mainstay of
treatment
60. In making objectives for health education, the following is important:
A. They are worded in terms of learner behavior
B. An affective objective requires hands-on experience
C. Evaluation clarifies what need to be strengthened
D. Instructor ability is important in planning the learning activity.
61. Partnership approach to community health development is among
the:
A. private sector, government sector and the community
B. community and its local government leaders
C. international and national agencies
D. private and government practitioners
62. The
the
A.
B.

first contact of the community to the health chain, as defined by


PHC system is the:
village health workers
C. hospital personnel
intermediate level health
D. barangay captain
workers

63. The health status of a community may be measured or expressed in


terms of:
A. its statistical indices of morbidity and mortality
B. availability of health care services
C. availability of public utilities
D. utilization of health care services
64. Community planning should start with:
A. formation of objectives
B. identification of roles and their relationships
C. educational analysis
D. determination of resources
65. Identifying and prioritizing health problems is part of this planning
process:
A. situational analysis
C. plan investigation
B. plan implementation
D. plan formulation
66. Best people to identify and prioritize community health problems:
A. community residents
C. health workers
B. social workers
D. consultants
67. The initial step in planning for the provision of health and medical
care services for a community is to:
A. define the health problems
C. set the objectives
B. determine the projected
D. decide on what services to
budge
provide

68. The gross domestic product adjusted with the net factor income from
the rest of the world:
A. Gross National Product
C. Gross Domestic Product
B. Gross Value Added
D. Consumer Price Index
69. The objectives in the tertiary prevention of non-communicable
diseases
include the following:
A. help the patient function maximally within the restrictions
imposed by the disease
B. removal of the agent from the environment
C. screening
D. immunization
70. In computing the cost of illness, the following data are needed:
A. Average daily income or wage
B. Average days of non-morbid condition
C. Average cost of savings
D. Number of death
71. Health education involves:
A. process of translating knowledge into action
B. telling people what to do
C. giving lectures about health
D. issuing rules and regulations to stop a bad habit
72. The primary objective of health education is to:
A. improve health practices
B. transfer technology to lay personnel
C. impart knowledge
D. promote health in general
73. Public health services (national and local) must give emphasis on:
A. prevention, promotion and maintenance of health
B. treatment of diseases
C. rehabilitation of the disabled
D. provision of emergency and first aid services
74. The role of government agencies in solving community health
problems:
A. guide, assist and facilitate community efforts in meeting its needs
B identify the problem of the community
C. provide funds for the project
D. provide comparative statistics
75. In public health, the most frequent problem of scarcity of resources is
best met by:
A allocate resources according to needs
B. transfer of funds
C. complete for national funds
D. increase manpower training
76. The managerial approach to motivation in health development is:
A. to develop a shared responsibility for achieving organizational
and individual goals by contributing on the basis of his interest
and abilities
B. manipulate workers by considerate treatment

C. induce workers to perform by giving them high wages


D. motivate workers to do overtime work
77. The most effective change in health behavior occurs when:
A. the individual incorporates the change into his system of values
B. the educator is an attractive figure
C. a reward is offered if the change is adopted
D. the individual is threatened with punishment if he does not
accept the
Change
78. Evaluation of a health program is necessary in order to:
A. determine if other objectives have been attained
B. prematurely stop the program implementation
C. allocate available resources to another program
D. re-assign health manpower to other worthy programs
79. The basic indicator most often used to reflect the economic health of
a
country is:
A. infant mortality rate
C. migration rate
B. infant mortality rate
D. population density
HEALTH CARE: PHILIPPINE SETTING
80. The most common type of attendant at birth utilized by Filipino
women:
A. Hilots
C. physicians
B. Midwives
D. nurses
81. Maternal health status of the Philippines is still poor because of the
following:
A. doctors attend to of deliveries
B. birth rate is relatively low
C. maternal death rate is relatively low
D. deliveries are in hospital
82. In the Philippines, the primary factor to consider in the design of the
health care delivery system is:
A. prevalent diseases
C. education of the masses
B. population growth
D. nutritional requirement
83. Republic Act 7875 is also known as The National Insurance Act of
1995
covers:
A. all Filipino citizens
C. government employees only
B. private employees only
D. unemployed citizens
84. The priority or target group in the National Anti-TB Program as
recognized by PHILHEALTH and DOH is:
A. sputum positive cases
B. sputum negative but with history suggestive of TB
C. sputum negative cases but with possible X-ray findings
D. sputum negative but with clinical findings suggestive of TB
85. This specifically consists of a core list of drugs approved and
authorized by the DOH:
A. Essential Drug List
C. Positive Drug List

10

B. Generic List

D. Prohibited Drug List

86. By tailored procurement of drug by government, it will make available


to
its own clientele:
A. the best drug at least cost
C. bulk buying
B. the expensive drugs at low D. the cheapest drug
cost
87. DOH Office that is responsible for handling matters pertaining to
drugs,
laboratories, facility and professional licensing is:
A. Standards and regulations C. Hospital services
B. Management services
D. Public Health services
88. DOHs goal of improving the general health status of the population
require:
A. Reduce infant/child mortality rate C. Increase total fertility rate
B. Decrease life expectancy
D. Increase growth rate
89. PHILHEALTH Benefits Package includes:
A. Normal Spontaneous Delivery of first two (2) births
B. Outpatient psychotherapy and counseling for mental disorder
C. Drugs and alcohol abuse and dependency treatment
D. Home and rehabilitation services
90. The primary aim of decentralization is:
A. To increase resource base for primary care, shifting from central
to peripheral
B. To decrease health center utilization rate
C. To give accountability to political authority
D. To relegate procurement of drugs and supplies to LGUs
91. Prevention of cancer recurrence and complications falls under what
level of prevention of the DOH Cancer Control Program:
A. Tertiary
C. Primary
B. Secondary
D. Primordial
92. True of TB in the Philippines:
A. DOTS aims to improve treatment compliance
B. More common in age group 60 and over
C. Three times more common among females than males
D. Remains the # 1 leading cause of mortality in the country
93. The most effective control measure for Tuberculosis which is
prevalent in the Philippines is:
A. Mass BCG vaccination
B. Clean Air Campaign to eliminate pollution
C. Massive nutritional correction and support
D. Promotion of chemoprophylaxis for contacts/exposed population
94. Which of following tops the list of major causes of intestinal
parasitism in the Philippines:
A. Ascaris Lumbricoides
C. Capillaria Philippinensis
B. Hookworms
D. Trichuris Trichiura

11

95. Which of the following has the highest prevalence of all the risk
factors to cardiovascular diseases according to the DOH
Cardiovascular Protection Program?
A. hypertension
C. obesity
B . smoking
D. stress
D. Bereavement must cater to the family members also
96. In 2003, National Nutrition Survey (FNRI) showed that the prevalence
of this vitamin and mineral deficiency remains high for:
A. Vitamin A & iodine
C. Vitamin A & Zinc
B. Vitamin B & iron
D. Vitamin D & iodine
SIMPLE MULTIPLE CHOICE. CHOOSE THE BEST ANSWER.
GOVERNMENT PROGRAMS
97.If both parent are Medicare members, the dependent children
benefit from membership of:
a. Mother
c. Father
b. Both mother and father
d. One can choose

may claim

98.Family planning means:


a. Voluntary descisions and positive action of couples to have a desired
number of children
b. Intra-uterine device and pills
c. Legal abortions
d. Knowledge of the different family planning methods
99.The major activities undertaken in the maternal and child health program
does NOT include
a. Immunization of pregnant mothers against measles
b. Proper pneumonia prevention
c.
Promotion of prenatal care
d. Monitoring of growth and health status of infants and children
100.
In the Under Five Program, the best screening procedure for
tuberculosis:
a. Sputum examination
b. Surveillance
c. Tuberculin testing
d. Chest x-ray
101.

The prerequisite for any rational drug policy is


A. Availability
B. Accessibility
C. Quality assurance
D. Affordability

102.
Drugs not covered by patent protection and which are labeled solely by
their international proprietary name is:
A. Complementary drugs
B. Essential drugs
C. Generic drugs
D. Core drugs
103.
A.
B.
C.
D.

PhilHealth gives medical coverage to dependents of members who are:


parents over 60, children below 21
parents over 65, children below 21
parents over 60, children below 18
parents over 65, children of any age

12

104.
Philhealth benefits vary according to the following except:
A. Relative unit value of surgery
C. Type of hospital
B. Actual cost of care
D. Type of physician
105.
Phil-Health sets standards, guidelines and procedures prior to allowing
doctors and hospitals to become part of the NHIP. This process is called:
A. evaluation
C. recognition
B. accreditation
D. acceleration
106.
a.
b.
c.
d.

The Family Planning program consist of the foloowing EXCEPT:


Promotion of knowledge on the use of contraceptive devices
Helping childless couples to bear children
Encouraging legal abortion
Providing family planning services

107.
The following preventive measures are included in the National TB
Control Prgram EXCEPT:
a. Giving chemoprophylaxis for those not yet infected
b. BCG vaccination of eligible population
c. Early treatment of infective TB cases
d. Health education
108.
According to the revised Expanded Program of Immunization, the
contraindication/s to immunization is/are:
a. Malnourished child
c. Diarrhea
b. Temperature of 37.5 38o
d. Convulsions after DPT
109.
A greater measure of success of a TB control program could be
expected if activities were centered on:
a. Wider coverage of BCG immunization
b. Development of more effective treatment methods
c. Amelloration of the socio-economic status of the general population
d. Identification and treatment of early cases
110.

Toxic effects of INH has been reported as:


a. Diarrhea
c. Chorioretinitis
b. Hepatitis
d. Extrapyramidal symptoms

111.
In an under six program the best screening procedure
tuberculosis is:
a. Chest x-ray
c. Tuberculin test
b. Sputum examination for AFB
d. B and C only
112.

All are ingredients of Oresol (oral rehydration fluid) EXCEPT:


a. Calcium chloride
c. Potassium chloride
b. Sodium chloride
d. Glucose

113.

Routine immunization of children include the following EXCEPT:


a. BCG
c. Measles
b. Thyphoid
d. Diphtheria

for

114.
The folowing immunizations are recommended for 3-months old
children consulting at an under six clinic:
a. DPT
d. A and B
b. Measles vaccine
e. A and C
c. BCG
115.
Immunization is contraindicated among
a. Malnourished children
c. Children with diarrhea
b. Breastfed babies
d.
Children with prolonged febrile
illness

13

116.
Contraindications for administering a live attenuated vaccine include
all of the following EXCEPT:
a. Acute febrile illness
b. Recent administration of immune globulin host
c. Immunosuppressive disorder or compromise
d. Administration of another live vaccine
117.

BCG vaccination is administered on the right deltoid are a by:


a. Intramuscular method
c. Intradermal method
b. Subcutaneous method
d. Scratch method

118.

The recommended WHO schedule for measles vaccination is at:


a. Birth
c. 3 months
b. 6 weeks
d. 9months

119.
Maternal antibodies convey passive measles immunity to infants
probably until age:
a. 12 to 15 months
c. 3 to 6 months
b. 6 to 9 months
d. 18 24 months
120.
The descision to give the rabies vaccine is influenced by any of the
following EXCEPT:
a. The severity of the bite
b. The condition of the animal at the time of exposure
c. The part of the body that was bitten
d. The condition of the animal 15 days after axposure
121.
The following are the statements of Policy of the Generic Act of 1988,
EXCEPT:
a. Promote, encourage, and require the use of generic terminology
b. Penalize any violation of the Act
c. Emphasize the scientific basis for the use of drugs
d. Ensure adequate supply of drugs with generic names
122.
The following organizations jointly with the Department of Health are
involved in the education drive for the Generics Acts, EXCEPT:
a. Department of Finance
b. Philippine Information Agency
c. Department of Local Governments
d. Department of Education, Culture & Sports
123.
Which of the folowing would be best reflect the inadequancy of
maternity care services in the community?
a. Small proprotion of hospital births
b. Small proportion of deliveries attended by physicians
c. Large proprotion of maternal deaths due to hemorrhage
d. None of the above
124.

The leading cause of maternal death in the Philippines is:


a. Abortion
c. Toxemia of pregnancy
b. Infection
d. Hemorrhage

125.
a.
b.
c.
d.
126.

High risk in maternal mortality are mothers


Under 15 years or above 45 years
17 20 years
20 25 years
30 35 yers

The most effective method of contraception is:


a. IUD
c. Rhythm method
b. Pills
d. Barrier or condoms

14

127.
a.
b.
c.
d.

This is NOT one of the priority needs of a mother


Nutrition promotion and growth surveillance
Control of infectious disease including tetanus
Maternal care
Nutrition supplementation

128.

The target group for tetanus toxoid immunization is:


a. Women 12-59 years
c. All persons
b. Women 15-44 years
d. All women regardless of age

129.

At birth a Filipino baby usually weighs:


a. 2000 gms
c. 3000 gms
b. 2500 gms
d. 4000 gms

130.
a.
b.
c.
d.
131.
many
a.
b.
c.
d.

Prevention of prematurity can be done by:


Promotion of preconceptional care
Improvement of obstetrical service
Improvement of maternal nutrition
Improvement of prenatal care
Among the following factors, which has the LEAST influence on the
infants during the first day of life?
Poor environmental sanitation
Incompetent attendance at delivery
Inadequate prenatal care
Maternal malnutrition

132.
Chemotherapeutic agents are applied to the eyes of the newborn to
prevent
a. Juvenile cataract
c. Syphilis
b. Gonorrhea
d. Trachoma
PUBLIC HEALTH
133.
Long-standing lack of vitamin A is maifested in the form of this ocular
lesion:
a. Keratomalacia
c. Nyctalopia
b. Corneal xerosis
d. Bitots spots
134.
If a 6-month old baby is suffering from diarrhea, the following should
be told to the mother:
a. Stop breast-feeding
c. Give am and tea only
b. Stop the usual food given
d. None of these
PUBLIC ADMINISTRATION
135.
The role of governmental agencies in solving community health
problems is to:
a. Identify the problem of the community
b. Provide funds for the project
c. Provide comparative statistics
d. Guide, assist, and facilitates community efforts in meeting its needs
136.
In public health, the most frequent problem is scarcity of resources.
This can be best met by:
a. Transfer of funds
c.
Allocate resources according to
needs
b. Compete for national funds
d. Increase manpower training
EPIDEMIOLOGY

15

137.
likely
a.
b.

In writing a research proposal, the formal statement about the most


outcome of the proposed research falls under:
Literature review
c. Hypothesis
Methodology
d. Objectives

138.
The factors that effect the reliability of an instrument are the following
EXCEPT:
a. Observer variation
c. Lack of congruency
b. Lack of precision
d. Lack of validity
139.

A control or comparison group is necessary in:


a. Descriptive studies
c. Cross-sectional studies
b. Prevalence studies
d. Prospective studies

140.
The importance of reviewing the available literature related to research
problem is
a. To prevent duplication of work that has been done before
b. It may lead to refine the problem definition
c. To find out the authors of these works
d. To provide good reasons for others to support the proposed research
141.
Among the following demographic characteristics, which does not
contribute to high fertility in the Philippines?
a. High marriage rate
c. Increasing life span
b. Young population
d. Low median age on marriage
142.

The greatest contributor to population growth in ASIA today is:


a. Increased mortality rate
c. Decreased birth rate
b. Increased birth rate
d. Decreased mortality rate

143.

Life expectancy is lower among the


a. Married
c. Young professionals
b. Single
d. Separated / divorced

144.
is the
a.
b.
c.
d.

In the choice of a problem for research, which of the following criteria


LEAST important?
Availability of technical and logistic support
Probability of solving the problem by research
Probability of acceptance for publication
Useful applicability of the results

145.
Because of its inherent weakness, this method is not utilized in the
collection of scientific data:
a. Census
c. Registration method
b. Questionnaire method
d. Case record method
146.
The following epidemiologic approach may be utilized when the
disease being investigated has not been studied yet in the past:
a. Descriptive
c. Cohort prospective
b. Retrospective
d. Experimental
147.
This type of study can best demonstrate a cause and effect
relationship:
a. Experimental study
c. Analytic study
b. Descriptive study
d. Cross-sectional
148.

The possibility of cause-effect relationship is difficult to assess in:


a. Experimental
c. Cohort
b. Cross-sectional
d. Case-control

16

149.
To determine if there is any relationship between oral contraceptive
use and chlamydia infection, an investigator selected a sample of 100
women attending a social hygiene clinic. He then measured the predictor and
outcome variables by talking a history of oral contraceptive use and sending
a cervical swab to the lab for chlamydia culture. The design of this study is:
a. Cohort study
c. Case control
b. Cross-sectional
d. Experimental study
150.
a.
b.
c.
d.

What is the first step in the conduct of a clinical trial?


Clearly and precisely state the reasons for conducting the trial
Clearly define the criteria for selecting the participants
Clearly precisely state the objective/s of the trial
Make a clear staements of facts governing conduct of the trial of the
would be participants

151.
The observational study in which subjectys are sampled based on the
presence or absence of a risk factor interest, these subjects are followed over
the time for the development of a disease outcome:
a. Case control study
c. Cohort study
b. Ecologic study
d. Cross-sectional study
152.
An investigator wanted to determine if there is a significant difference
in the mean systolic blood pressure 10 women before and after oral
contraceptive use for 3 months. The appropriate statistical test for
significance for this study is:
a. Analysis of veriance
c. Student t-test
b. Chi-square analysis
d. Palred t-test
153.
a.
b.
c.
d.

In chi-square, one would like to demonstrate that


There is a definite cause-effect association
There is a significant association among categories
There is a significant correlation
The number in the 2 x 2 table are diffecrent from one another

154.

The objective of data processing is to facilitate


a. Statistical analysis
c. Completion of data
b. Termination of research
d. Summarizing of data

155.

Descriptive statistics include the following EXCEPT:


a. specific rate
c. graphical representation
b. measures of dispersion
d. tabular presentation

156.
value
a.
b.

The measure that tends to be misleading in the presence of erratic


is
Mode
c. Median
Mean
d. All of the above

157.
When distribution is symmetrical, the manner of tendency what should
be chosen is:
a. Mode
c. Mean
b. All of these
d. Median
158.
Randomization is a procedure used for assignments or allocation of
subjects to treatment and control groups is experimental studies.
Randomization ensures
a. That assignments occurs by chance
b. That treatment and control groups are alike in all respects expect
treatment
c. Bias in observation is eliminated
d. The placebo effects are eliminated

17

159.

When samples are chosen because they are handy and easy this is:
a. Simple random sampling
c. Haphazard sampling
b. Systematic sampling
d. Cluster sampling

160.
One of these is NOT included in the comprehensive maternal and child
health program policies:
a. Pregnant women should be given iron tablets during the second and
third trimester and throughout the lactation period
b. Abnormal pregnancies may be delivered at home by a professionallytrained health worker
c. Growth and development of infants and children should be monitored
by growth charts and developmental screening devices
d. The newborn receive immediate care at birth and breast feeding
instuted within 30 minutes after delivery
161.
Population density is determined by use of two data. One is total
population and the other is:
a. Age and sex distribution of the community
b. Area of the community
c. Income of the community
d. Resources in the community
162.
a.
b.
c.
d.

Population would become younger if:


Both its crude birth rates and crude death rates remain high
Its crude birth rates remain high but its crude death rates decline
Its crude birth rates decline but its crude death rates remain high
Both its crude rates and death rates decline

163.
The population pyramid shows the distribution of a population
according to:
a. Age and sex
c. Fertility
b. Inflow-outflow equation
d. Percentage of births and deaths
164.

The arrangement of the population in space in a given time


a. Composition
c. Spatal distribution
b. Social mobility
d. Size

POISONING
165.
In childhood poisoning, emesis should NOT be induced if the poison is:
a. Barbiturates
c. Drain cleaner
b. Acetaminophen
d. INH
166.
The following does NOT conform to the 1987 Family Code of the
Philippines
a. The contracting parties to a marriage maybe of the same gender
b. Marriage solemnized by a person not legally authorized to perform
marriage are void
c. A marriage contracted by any person during the subsistence of a
previous marriage shall be null and void
d. Mayor can no longer solemnize marriage
RATES AND RATIO
167.
The current priority health problem of the Philippines is:
a. Malnutrition
c. Rapid population growth
b. Communicable disease
d. All of the above
168.
The office that is changed with the function of collecting and reporting
of vital health statistics is the:
a. Bureau of Census and Statistics c. Health Information Service DOH
b. Bureau of Medical Services
d.
Department of Disease
Control

18

169.
This is the measure of mortality which will tell us what the 10 leading
causes of death rate are:
a. Case fatality rate
c. Infant mortality rate
b. Cause specific death rate
d. Proportionate death rate
170.
a.
b.
c.
d.

The purpose of immediate notification of disease is to:


Study the natural cause of the disease
Update the statistical data
Prevent the spread of the disease
Study the trend of the disease

OCCUPATIONAL HEALTH
171.
One of the following cancers is NOT associated with occupational
exposures:
a. Breast CA
c. Lung CA
b. Bladder CA
d. Liver CA
172.
The conservation of health in relation to work, the
environment and assurance of work efficiency is:
a. Industrial hygiene
c. Occupational medicine
b. Occupational health
d. Industrial medicine
173.

working

Unwanted sound has this physiologic effect on an employee:


a. Deafness
c. Inefficiency
b. Irritation
d. Autism

ETHICS
174.
Code
a.
b.

The primary objective of the practice of medicine according to the


of Medical Ethics in the Philippines is:
Be good citizen
c. Be solicitous to patients
Be a fiend of man
d. Service to mankind

FAMILY MEDICINE
175.
The following are important roles of a family physician:
a. Personal physician to each member of the family
b. Manages the collective health problems of the family
c. Recognizes the effects of illness upon the other family members
d. All of the above
176.
a.
b.
c.
d.

The family life cycle:


Depicts the expansion and correction of the family
Focuses on major events of developments within a single family
Involves a sequence of stressful changes
A and B only

177.
Measures that the family physician can take to reduce the impact
illness on the family are as follows:
a. Do not include the family in deciding on the care of the sick
b. Inform the family of measures important to the success of treatment
c. Watch for change in attitude or negative reaction among family
d. A and B only
e. B and C only
178.
a.
b.
c.
d.
e.

The Philippine age structure is considered young. It means


A big percentage of the population is made up of the youth
More dependents for workers
People are retiring young
A and B only
B and C only

19

PRIMARY HEALTH CARE


179.
The major cause of disabilities in developing countries is:
a. Inadequate nutrition
c. Accidents
b. Infectious disease
d. A and B
180.
The first contact of the community to the health chain, as defined by
the PHC system is the:
a.
hospital personnel
c. intermediate level health
b.
village health workers
d. barangay captain workers
181.
The following are true for volunteer community health workers,
EXCEPT:
a. They establish linkage between government and non-government
organization
b. They are residents of the community
c. they are community-based
d. they provide only curative care
182.
When
defining the
known as:
a.
b.

a community shares in the responsibility and participates in


health and health-related problems in the community, this is
intrasectoral linkage
c. appropriate technology
community participation d. intersectoral linkage

183.
Which of the following is NOT included in the essential health service of
Primary Health Care?
a.
immunization against the major infectious diseases
b.
provision of essential drugs
c.
provision of safe water and basic sanitation
d.
appropriate treatment of all existing diseases
184.
Care?
a.
b.
c.

Which of the following is in line with the principles of Primary Health

doctors make all the decisions in a community health program


free clinics are held in the population
a barangay health council is established to plan, implement and
evaluate community program
d. hilots are not allowed to attend deliveries

185.
Primary Health Care (PHC) refers to
a. the first contact of a patient with a professional health care provider
b. a sub-system of the health care delivery system
c. a type of health care program designed for communities
D. an approach to making health care available and accessible to the
population
186.
A health care system that has the greatest impact on the health of a
society, while making the best use of its resources, fulfills which of the
following values of social accountability?
A. Relevance
C. Cost-effectiveness
B. Quality
D. Equity
187.
Parents who are not qualified as legal dependents, indigents or retirees
can avail of PhilHeatlth benefits through the:
A. Medicare Para Sa Masa
C. Sponsored Projects
B. Individually-paying program
D. Non-paying members
188.
An important feature of a Community-Based Health Program is:
A. preset program objectives
B. managed solely by the peoples organization

20

C. people participation in all stages of development


D. availability of modern health equipment for community
Answer

questions

page

1.

An act instituting a national health insurance program for all Filipinos and
establishing the Philippines Health Insurance Corporation.
A. Republic Act 7719
B. Republic Act 7875
C. Republic Act 7160
D. Republic Act 6675
MPL: 1

386

This is also known as Clean Air Act.


A. Republic Act 7277
B. Republic Act 7883
C. Republic Act 8749
D. Republic Act 7600
MPL: 1

389

House Bill 618 is also known as:


A. National Tobacco Control Act
B. Rooming-In and Breastfeeding Act
C. Generics Act
D. National Blood Services Act
MPL: 1

403

The National Health Insurance Program (NHIP) shall give the highest priority
to achieving coverage of the entire population with at least a basic minimum
package of health insurance benefits, the guiding principle is:
A. Social solidarity
B. Equity
C. Compulsory coverage
D. Universality
MPL: 1

409

Mandates iodization of all food grade salt and making this available to all
communities nationwide.
A. Republic Act 8423
B. Republic Act 8976
C. Republic Act 8172
D. Republic Act 7876
MPL: 1

385

Children were hospitalized after their mother fed them with poisonous mushroom.
The mother was unable to differentiate poisonous from nonpoisonous species.
How would you assess the Meffect of her action?
A. Directly voluntary
B. Indirectly voluntary
C. Positively voluntary
D. Negatively voluntary
MPL: 0.5

A patient who has emphysema finds it difficult to quit smoking. How would you
assess the morality of the patients action?
A. Imperfectly voluntary because of impairment of knowledge
B. Imperfectly voluntary because of habit
C. Perfectly voluntary act
D. An act of man not performed without free will
MPL: 0.5

Which among the following is a human act?


A. Cheating in class
B. Killing by a known schizophrenic
C. An accident caused by severely intoxicated person
D. Killing of a wife by the husband who caught her in the act of infidelity
MPL: 0.25

21

A superior officer not stopping an evil act of a lower rank officer despite knowledge
of the same is _________.
A. Equally guilty as that of the lower rank officer
B. Not guilty of the act
C. Guilty but of a lesser degree
D. Not morally accountable
MPL: 0.5

10

Withdrawing food and water in a progressively deteriorating Alzheimer disease


patient. Which principle is being violated?
A. Beneficence
B. Non-maleficence
C. Autonomy
D. Justice
MPL: 0.5

11

Which among the following situations is ethical?


A. An HIV positive patient continues to engage in unprotected sex.
B. Commercial sex workers are mandated to undergo regular screening for sexually
transmitted diseases including AIDS.
C. Donating blood for a fee
D. The owner of an establishment is doing a research on smoking. He asked all
his employees to be part of the study.
MPL: 0.25

12

A 75-y/o diabetic patient with gangrenous foot refused to be amputated knowing


all the risks and benefits. She said she is too old to live without a leg. The doctor
agreed. What is your ethical evaluation of the doctors decision?
A. Right. Patients autonomy is respected.
B. Wrong. Theres harm to the patient.
C. Right. Patient accepted an additional burden.
D. Wrong. Patients ability to decide is questionable.
MPL: 0.5

13

A patient told his doctor that he do whatever is best for him. This is not a violation
of the principle of autonomy because of the following reason:
A. Accepts an additional burden
B. Gives up what is due
C. Delegate authority
D. Looses right to what is due
MPL: 0.25

14

Principle that emphasizes the freedom to have children


A. Autonomy
B. Inviolability of life
C. Beneficence
D. Stewardship
MPL: 1

15

The principle of beneficence means


A. To do good
B. To do good at all times
C. To do good without any harm
D. To do good with minimal harm
MPL: 0.5

16

The following are true EXCEPT


A. Truth telling of harm overrides confidentiality
B. In emergency situation informed consent can be waived
C. Unplugging of life-sustaining machines is justifiable if and when they are no
longer useful to the dying person
D. Clinical research need not involve animal experimentation
MPL: 0.25

17

A 70-y/o quadriplegic patient (paralyzed in all 4 limbs) following repeated


strokes requests that no resuscitation be performed if he goes into cardiac
arrest. The following would justify a Do Not Resuscitate order EXCEPT

22

A. Resuscitation will cause serious physical and emotional burden


B. Resuscitation is against patients autonomous wish
C. Resuscitation is extraordinary means
D. Resuscitation is proportionate means
MPL: 0.25
A

18

The principle of totality is the governing principle in the following situation:


A. Plastic surgery and mutilation with due cause
B. Voluntary organ donation
C. Confinement of mentally against their will
D. Conflict of interest
MPL: 0.5

19

Hospitals cannot be held liable for patients who go home against medical advice
because patient
A. Gave up what is due
B. Lost right to what is due
C. Accepted an additional burden
D. Delegated authority
MPL: 0.5

20

A patient in the ward has renal failure. As a junior intern assigned to monitor
the patient, you noticed that his medication is being given in toxic dose. Which
virtue is needed in this situation?
A. Integrity
B. Courage
C. Respect
D. Honesty
MPL: 1

21

Which among the following situations is a nonviolation of the principle of justice?


A. Kidneys for transplant are scarce. You decided to buy from indigent patients.
B. Allocating more of the government funds to kidney transplant program
than to the free immunization program for children under six
C. Following a typhoon, health care is given to the victim who will die without an aid
D. Compulsory tubal ligation in government hospital
MPL: 0.5

22

Which among the following patients vying for one available respirator would gain
the most from using the respirator?
A. Trauma victim with severe brain damage. Only few brainstem functions
remain. He is unlikely to recover.
B. Patient with infection of the nervous system that rendered him paralyzed from
the trunk down. He is, however, progressively improving and is expected to recover.
C. Patient with complete and irreversible spinal injury that has left her paralyzed
from the neck down. She is able to talk but totally dependent on the respirator.
D. Cancer patient undergoing palliative treatment
MPL: 0.25

23

A poor patient needing care agrees to enroll in a research study so as to be


admitted in the hospital for free treatment of his medical problem. Was an
informed consent obtained in this case?
A. Yes. Information is understood by subject. No evidence of constraint on
subjects voluntariness.
B. No. Lack of competence
C. No. Questionable voluntariness due to subjects personal circumstance.
D. No. Probable incomplete disclosure and deception.
MPL: 0.5

24

An employee was discovered to be sick during the annual physical examination


sponsored by the company. His health would adversely be affected if he
continues with his job. He pleads not to tell his employer. What should the physician do?
A. Maintain confidentiality at all times
B. Break confidentiality for the best interest of the patient

23

C. Break confidentiality for the best interest of innocent third party


D. Wait and observe
MPL: 0.5
A

25

Which among the following is NOT a condition to stop treatment in a dying patient?
A. Life is preserved by ordinary means
B. Patient and/or family consents
C. Irrefutable evidence that biological death is imminent
D. Treatment will not prolong life for any significant time
MPL: 0.25

26

The type of domestic violence, which is considered to be the most common of elder
mistreatment, is called
A. Physical abuse
B. Material exploitation
C. Caregiver neglect
D. Emotional abuse
MPL: 1

27

An elderly patient was noted to have hematoma in the legs. Upon probing, patient
claimed he fell from the bed. Caregiver was hesitant to bring him to the hospital for
medical management. What will you do?
A. Report to authorities
B. Perform home safety assessment
C. Expand social services
D. Assess family functioning
MPL: 0.5

28

Given a patient who is a victim of partner abuse, the following may be done EXCEPT
A. Draw a safety plan
B. Do couple counseling
C. Refer to mental health professional
D. Refer to womens desk
MPL: 0.5

29

In cases of sexual assault, the following is recommended:


A. Collect clothing and place in plastic bag
B. Do a urine pregnancy test
C. Give empiric antibiotic treatment for STD
D. Insert IUD within 24 hours to prevent pregnancy
MPL: 0.25

30

Tertiary prevention for those who are victims of violence includes


A. Screen for possible violence
B. Provide appropriate medical care
C. Inform women of their rights
D. Refer to local support group and shelter
MPL: 1

31

Family, as a group of people, is defined in various ways. Which among the following
description runs common among the various definitions?
A. Related by blood or marriage
B. Related by strong affection
C. Comprises a permanent household
D. Changes through time
MPL: 0.25

32

Tom and Tina have been living with their aunt and grandmother since their
parents went abroad to work as nurses. What is the type of family structure?
A. Nuclear
B. Extended
C. Single-parent
D. Communal
MPL: 1

33

As parents, Jim and Lora make sure that they spend quality time with their children.
They play and do things together with them. What function of the family do they
perform?

24

A. Biologic
B. Psychologic
C. Educational
D. Socio-cultural
MPL: 0.75
C

34

John and Marsha is a family with four children aged 16, 10, 6 and 4. Both
husband and wife are in their mid-forties. What second order changes can be
anticipated in their family life cycle stage?
A. Taking on parenting role
B. Sharing responsibilities with extended family
C. Focusing on marital and career issues
D. Keeping communication system open
MP: 0.25

35

Linda was diagnosed with myoma last 2004. Recently, she developed vaginal
bleeding. She was told she needs to undergo hysterectomy. She was subsequently
observed to be crying and socially withdrawn. Identify the stage in the illness trajectory.
A. Reaction to diagnosis
B. Major therapeutic efforts
C. Early adjustment to outcome
D. Adjustment to permanency of outcome
MPL: 1

36

Which among the following statements is NOT true?


A. For every disease, there is a corresponding impact of illness.
B. For illness with acute onset, the family is more prone to a crisis situation.
C. For stages 2 & 5 of the illness trajectory, the family will go through the same
process of denial, anger, and depression.
D. For chronic illness, the family is less prone to stress due to coping through time.
MPL: 0.25

37

Mrs. D began having difficulty remembering things, missing at time of work, and
appearing in public in an untidy state at age of 64 years. Mrs. D was brought to a
doctor for consult and was diagnosed to have Alzheimers disease. Because
Mrs. Ds condition seemed to progress rapidly, the attending physician did a home
visit. When he arrived, she found Mrs. D unkempt, bedridden and has lost weight.
The daughter who lives with her complained of exhaustion and of getting very little
help from her siblings. In assessing the condition of the index patient and the family,
the doctor made use of several tools. Which among the following tools and their
corresponding indication for use is NOT correct?
A. APGAR assess familial resources
B. Genogram identify alternate caregivers
C. SCREEM assess capacity of family to participate in health care
D. Family map describe family dynamics
MPL: 1

38

A 42-y/o woman, married with 2 daughters from Masbate revealed a 2-yr. history of
breast mass. Physical examination showed a cachectic patient with unilateral multiple
beast mass with foul smelling discharge. The patient sought consult with faith healers
in their hometown. Her family and relatives firmly believe that it was due to barang
(witchcraft). She was brought to Manila to consult with another faith healer. She
refused to see a doctor, as it would render ineffective the treatment being done to her
by the faith healer. Deep in her heart she has given up hope for cure. For a doctor to
understand the health beliefs and practices of the family and the index patient, what

25

tool in family assessment should be used?


A. Clinical Biographies
B. Family Circle
C. Ecomap
D. SCREEM
MPL: 1
C

39

In evaluating the potential crisis once the patient dies, which among the following
would help evaluate the degree of disruption in the function of the family?
A. Family stressor
B. Family developmental stage
C. Family role
D. Family coping history
MPL: 0.25

40

What would consists tertiary level of prevention for the family?


A. Hospice care
B. Counseling
C. Health education
D. Breast examination
MPL: 1

41

It is essential health care based on practical, scientifically sound and socially


acceptable methods of technology made universally accessible to individuals
and families in the community and through their full participation and a cost that
the community and country can afford to maintain at every stage of their
development in the spirit of self reliance and self determination.
A. Primary Health Care
B. Primary Care
C. Secondary Care
D. Tertiary Care
MPL: 1

42

An evaluation or appraisal of present conditions and existing resources is


A. Planning
B. Assessment
C. Situational analysis
D. Implementation
MPL: 0.25

181

43

Demographic factors in planning includes description of


A. Existing health services
B. Working health systems
C. Socio-economic-environmental conditions
D. Age-gender composition and distribution
MPL: 1

181

44

Health status evaluation includes the following, except


A. Infant mortality rate
B. Hospital services utilization
C. All of the above
D. None of the above
MPL: 1

181

45

255

46

A gap between what is and what should be


A. Idea
B. Problem
C. Priority
D. Condition
MPL: 0.25
In the Primary Health Care approach, focus group discussion and community
assembly are strategies for:
A. Appropriate technology
B. Networking and linkages
C. Establishing support groups
D. Community participation

89

26

MPL: 0.25
B

47

Working together with the people, learning their language and eventually
189
establishing a herbal-medicinal garden for people consumption is an example of
A. Establishing support groups
B. Appropriate technology
C. Networking and linkages
D. Community participation
MPL: 0.25

48

An example of establishing support groups for program continuity


263
A. Committing civic, religious, non-government organizations to actively participate
B. Putting up a Botika sa Barangay with the help of the Municipal council
C. Creation of an organization of asthmatic patients in the early identification and
proper referral of asthmatics
D. Consultative meetings with community leaders for the creation of latrines in
certain areas
MPL: 0.25

49

Among the list, the program that would benefit the greatest number of people:
A. Provision of food and drugs
B. Accessibility to safe water
C. Health education
D. Proper waste collection and disposal
MPL: 0.25

305

50

In the course of implementation, when the quality of service is not at par,


management should:
A. Review established standards
B. Reprimand staff concerned
C. Measure difference in output
D. Modify activities/objectives
MPL: 0.33

344

51

During an evaluation procedure, complaints against nurses have been recorded


and analyzed. In making decisions, management should be reminded of:
A. Error of practical significance
B. Statistical data as evidence based materials
C. Opinions of co-managers and staff
D. Generalization based on records at hand
MPL: 0.25

342

52

Epidemiology can be defined as the study of:


A. The etiology of disease in humans
B. The frequency of causes of death in humans
C. The determinants of frequency pf disease in humans
D. The distribution and determinants of frequency of disease in human populations
MPL: 0.25

53

The time interval between entry of an infectious agent into a host and the onset
symptoms is called
A. The communicable period
B. The incubation period
C. The preinfectious period
D. The noncontagious period
MPL: 0.25

54

Primary prevention may be best undertaken during the period of:


A. Pre-pathogenesis
B. Pathogenesis
C. Resolution or sequelae
D. Any of the above
MPL: 0.33

55

In the study of the cause of a disease, the essential difference between an


experimental study and an observational study is that in the experimental
investigation

27

A. The study is prospective


B. The study and control groups are of equal size
C. The study and control groups are selected on the basis of history of exposure
to the suspected risk factor
D. The investigators apply an intervention to influence the outcome of the study, for
effective methods of treatment, prevention, or clinical management
MPL: 0.25
B

56

The occurrence of a group of illnesses of similar nature at a rate above the expected
number is called:
A. Hyperendemic
B. Epidemic
C. Endemic
D. Pandemic
MPL: 0.5

57

To determine whether maternal deficiency of folate is a cause of congenital defects


of the neural tube, the mothers of 100 newborns with congenital neural tube defects
and 200 newborns without congenital neural tube defects were questioned about intake
of multivitamins and folate during pregnancy. What type of study is this?
A. Clinical trial
B. Cross-sectional
C. Cohort
D. Case-control
MPL: 0.25

58

A study revealed an odds ratio of 3.35 (95 % Confidence Interval= 1.35-8.42)


associated with maternal deficiency of folate. If the study described is accurate,
which of the following statements is true?
A.
Results suggest that a baby whose mother had folate deficiency is about
3.35
times as likely to be born with congenital defects of the neural tube as a baby
whose mother did not have folate deficiency and the association is significant
(p<0.05).
B.
Results suggest that the odds of giving birth to newborns with congenital
defects
of the neural tube among mothers with folate deficiency is 3.35 as compared to
mothers who do not have folate deficiency and the association is significant (p<0.05).
C.
Results suggest that the risk of giving birth to newborns with congenital
defects
of the neural tube among mothers with folate deficiency is 3.35 times as
compared to mothers who do not have folate deficiency and the association
is significant (p<0.05).
D.
The results provide no evidence that maternal deficiency of folate is
associated with congenital defects of the neural tube in the offspring.
MPL: 0.25

Lou Stewells, a pioneer in the study of diarrheal disease, has developed a new diagnostic test for
cholera. When his agent is added to the stools, the organisms develop a characteristic ring
around them (He calls it the Ring-Around-the Cholera [RAC] test). He performs the test on 100
patients known to have cholera and 100 patients known not to have cholera with the following
results:
(+) RAC test
(-) RAC test
Total
A

59

Cholera
91
9
100

No Cholera
12
88
100

The sensitivity of RAC test


A. 91/[91+9] x100%= 91%
B. 88/[9+88]x100%= 91%
C. 91/[91+12]x100%= 88%
D. 88/[12+88]x100%= 88%
MPL: 0.25

28

During the investigation of an outbreak of food poisoning at a summer camp, food histories were obtained form all
campers as indicated in the table below.
Consumed Food
Food served

Ill

Hamburger
Potatoes
Ice cream
Chicken
Lemonade

6
7
8
13
2

Not
Ill
4
3
10
5
8

Did Not Consume Food

Specific Attack Rate

Ill

6/10 = 60%
7/10 = 70%
8/18 = 44%
13/18 = 72%
2/10 = 20%

4
2
2
1
4

Not
Ill
4
3
2
7
4

Specific
Attack Rate
4/8 = 50%
2/5 = 40%
2/4 = 50%
1/8 = 12.5%
4/8 = 50%

60

The incriminated food item is most likely to be:


A. Hamburger
B. Potatoes
C. Ice cream
D. Chicken
MPL: 0.25

61

The number of deaths from a given cause in a specified time period divided by
the total deaths in the same time period is:
A. Crude death rate
B. Indirect death rate
C. Case fatality rate
D. Proportionate mortality rate
MPL: 0.25

62

If the infant mortality of a community 68/1000 live births. This means:


A. 68 out of every 1000 babies born alive died during infancy
B. 68 babies died before reaching their first birthday
C. 68 out of 1000 persons in that community were infant deaths
D. 68% of all deaths were infants
MPL: 0.25

63

The best source of data or distribution according to age, sex and geographical
location is:
A. Reports of occurrence of notifiable diseases
B. Birth certificate
C. Census
D. Registries of certain disease
MPL: 0.5

64

Approval of research proposals using human subjects is done by:


A. Department chairman
B. Research Committee
C. Institutional review board
D. PRC
MPL: 0.5

65

Which of the following sampling method is appropriate when a population is not


widely spread geographically:
A. Random
B. Multi-stage
C. Stratified
D. Systematic
MPL: 0.33

66

In public health, the rate that are used to measure illness is known as:
A. Morbidity rate
B. Natality rate
C. Mortality rate
D. Specific rate
MPL: 0.33

68

Which among the following measures fertility:


A. Sex ratio

29

B. Crude birth rate


C. Maternal mortality rate
D. Life expectancy
MPL: 0.33
C

69

In the absence of an attending doctor at death, the responsibility of signing the death
certificate goes to:
A. Closest of kin
B. Nurses
C. Local Health Officer
D. Midwife
MPL: 1

70

Objectives of the research tell:


A. What variables are being considered
B. The affirmation of the hypothesis formulated
C. When the biostatistician must come in
D. How much funds to request
MPL: 0.33

71

The part which tells the reader what the different parts of the diagram stand
for is called:
A. Source
B. Title
C. Legend
D. Footnote
MPL: 0.5

72

The research design is essentially:


A. What the researcher will do to answer the research questions
B. How the research will collect the data
C. Both
D. Neither
MPL: 0.5

73

The index that measures the killing power of a disease is:


A. Prevalence rate
B. Case fatality rate
C. Infant mortality rate
D. Swaroops index
MPL: 1

74

In a small sample in which an immediate knowledge of variation is needed,


the measure used is:
A. Standard Deviation
B. Range
C. Variance
D. Quartile deviation
MPL: 0.33

75

Specifying the kind of subjects best suited to the research question and where
to recruit them, is the process of:
A. Exclusion criteria
B. Sampling
C. Randomization
D. Selection criteria
MPL: 0.5

76

The purpose of blinding in data collection is to achieve:


A. Accuracy
B. Validity
C. Precision
D. Objectivity
MPL: 0.33

77

The population pyramid shows the distribution of a population according to:


A. Age and sex
B. Inflow-outflow equation

30

C. Fertility
D. Percentage of births and deaths
MPL: 0.5
C

78

The computed collective characteristics derived from the entire population


are called:
A. Variables
B. Statistic
C. Parameters
D. Indicators
MPL: 0.5

79

The probability that a person has the disease given a positive test result is:
A. Negative predictive value of the test
B. Sensitivity of the test
C. Specificity of the test
D. Positive predictive value of the test
MPL: 0.5

80

The disease or condition which directly caused the death of a patient is:
A. Immediate cause
B. Underlying cause
C. Intervening, antecedent cause
D. None of the above
MPL: 0.33

81

Confidence limits are calculated using:


A. The mean and its standard error
B. The mean and the range
C. The median and its standard error
D. The mean and the range
MPL: 0.5

82

Randomization is a procedure used for assignments or allocation of subjects


to treatment and control groups in experimental studies. Randomization ensures:
A. That placebo effects all eliminated
B. That treatment and control groups are alike in all respects except treatment
C. That observation is eliminated
D. That assignment occurs by chance
MPL: 0.33

83

Post-neonatal mortality has been closely linked to:


A. Environmental factors
B. Maternal health prior to pregnancy
C. Events during delivery
D. Maternal health during pregnancy
MPL: 0.5

85

The branch of science that counts, measures and numerically states fact is:
A. Demography
B. Medical statistics
C. Statistics
D. Health statistics
MPL: 0.5

86

This is the measure of mortality which will tell us what the 10 leading causes of
death rate are:
A. Case fatality rate
B. Cause specific death rate
C. Infant mortality rate
D. Proportionate death rate
MPL: 0.5

87

The administrator of a day care center reports that a child under his care has
just been diagnosed as having meningitis. He is concerned about the other children
in the center and wants to know what to do. Which of the following must be known
in order that recommendation can be made?
A. Number of children in the center

31

B. Prevalence of meningitis in the community


C. Age range of the children
D. Number of employees in the center
MPL: 0.25
D

88

Vaccine Preventable diseases, EXCEPT:


A. Hepatitis B
B. Tuberculosis
C. Diphtheria
D. Filariasis
MPL: 1

89

The most common cause of cancer deaths among both men and women in the
country is:
A. Lung
B. Breast
C. Liver
D. Colo-rectal
MPL: 1

86

Mosquito-borne disease/s, EXCEPT:


A. Malaria
B. Dengue
C. Tuberculosis
D. Filariasis
MPL: 1

40

90

91

Food-borne and water-borne disease/s, EXCEPT:


A. Typhoid
B. Dengue
C. Hepatitis A
D. Cholera
MPL: 1

92

93

Based on IMCI recommendation, a child with pneumonia or dysentery should


return for follow up in:
A. 2 days
B. 5 days
C. 14 days
D. 30 days
MPL: 1
If a child is seen with pus draining from the ear and discharge is reported for
less than 14 days or with ear pain, diagnosis is likely:
A. Mastoiditis
B. Acute ear infection
C. Chronic ear infection
D. No ear infection
MPL: 1

94

A child having diarrhea was noted to be irritable with sunken eyes and slow
return of skin pinch is classified as:
A. No dehydration
B. Some dehydration
C. Severe dehydration
D. Very severe dehydration
MPL: 1

95

An approach whose aim is to improve the performance of the health service


delivery system I order to better meet peoples needs, consistent with the basic
values of quality, equity, relevance and cost-effectiveness.
A. Primary Health Care
B. Towards Unity For Health
C. Devolution
D. Decentralization
MPL: 0.5

96

Recommended initial work up when suspecting PTB:


A. Chest X-ay

34

59-62

32

B. PPD
C. Sputum AFB
D. Culture
MPL: 0.5
B

97

Suspended solid particles which are generated by condensation from gaseous


state, generally after volatilization from molten metals:
A. Gases
B. Fumes
C. Mists
D. Dust
MPL: 0.25

98

The most important parameter of drinking water quality is:


A. Bacteriological
B. Chemical
C. Physical
D. Organic constituents
MPL: 0.33

99

Minimata disease is due to:


A. Fluoride
B. Lead
C. Arsenic
D. Mercury
MPL: 0.33

100

A measure to terminate existence of a hazard through destruction with the aid of


physical, chemical or biological agent:
A. Treatment
B. Shielding
C. Isolation
D. Substitution
MPL: 0.5

39

REFERENCE
Philippine Health Care Factbook Vol. 1
References for questions 6-40
Alora, Angeles, BIOETHICS, NTTC-HP, UP-Manila, 1991
Monge, Michael, Ethical Practices in Health & Disease. Manila:
Sinag-Tala Publishers, Inc. 1994
Rakel, Robert, Textbook of Family Practice, Philadelphia: W.B. Saunders Company,
2002, pp 71-82
Primer on Primary Health Care
PHA202 module, Department of Health Policy and Administration, UP-College of Public
Health
Readings in Health System Management, 2nd edition
Ref. For questions 52-60
MacMahon, B, and Pugh, T, Epidemiology Principles and Methods, 2 nd edition
Ref for questions 61-87
Mendoza, OM et al, Foundations in Statistical Analysis for the Health Sciences,
Manila, DEBS,
CPH-UPM, 1997

Simple Multiple Choice: Choose the Single Best Answer


Recall:
1.
Adequate emergency exit is a provision of:
A. fundamental physiologic needs
B. fundamental psychologic needs

C. protection against contagion


D. protection against accidents

Recall
2.
Which of the following is a fundamental psychologic needs of man that has to be met by
a
33

standard house
A. protection from excessive noise
B. provision of adequate space
environment

C. provision of adequate privacy


D. provision of unpolluted

Recall
3.

Which of the following is considered a special dwelling unit?


A. Hotel/Motel
C. Seminaries
B. Inn
D. Dormitories

Recall
4.
Which of the following is the considered a most valuable component of refuse?
A. fruit peels
C. paper
B. tin can
D. cardboard
Recall
5.
For urban areas, the most ideal waste disposal system is.
A. sanitary pit privy
C. sewerage system
B. septic tank system

D. pour-lush latrine

Recall
6.
Which of the belongs to the Category II waste disposal unit?
A. Cistern flush toilet connected to a sewerage system
B. Pour-flush latrine
C. Bucket latrine
D. Bored hole latrine
Recall
7.
The most common cardiovascular disease that affect Filipinos is
A. Rheumatic Heart disease
B. CVA/Stroke
C. Ischemic Heart disease
D. Hypertensive Heart disease
Recall
8.
An exercise program to be effective, an optimal level of intensity must be achieved. This
is measured in terms of the:
A. capacity to push himself to the limits
C. ability to endure pain
B. maximum heart rate for age
D. threshold for exhaustion
Recall
9.
Which of the following anthropometric measurements is the best indicator of current or
recent malnutrition?
A. weight for age
C. weight for height for age
B. height for age
D. body mass index

Recall
10.
A short-term solution to micronutrient deficiency disorders is:
A. food fortification
C. tri-media nutrition education
B. supplementation
D. community food production
Analysis
34

11.

Which of the following will not be achieved by a totally successful Philippine Plan of
Action for Nutrition (PPAN)?
A. Full fortification of selected Philippine made food items with vitamin A , Iodine, and
Iron
B. Complete compliance and adoption of the Filipino family to desirable food and
feeding practices
C. Eradication of all causes of malnutrition in the Philippines
D. Assurance of food supply adequacy with augmentation from home and community
food production

Recall
12.
It is only the female Anopheles mosquito which is involved in the development of
Plasmodium specie because
A. there are no male Anopheles mosquitoes
B. female mosquitoes need a blood meal before they can produce eggs
C. male Anopheles have no proboscis
D. only the female mosquitoes are attracted to human blood
Recall
13.
The natural mode of transmission of malaria is by
A. co-natal transfusion
B. contaminated needles
C. inoculation of sporozoites through the mosquito bites
D. blood transfusion
Recall
14.
How many provinces in the country are endemic for malaria?
A. 65 of the 78 provinces
C. 12 of the 78 provinces
B. 40 of the 78 provinces
D. 8 of the 78 provinces
Recall
15.
The Expanded Program of Immunization (EPI) of the government is aimed at reducing
the Infant Mortality and Morbidity by decreasing the prevalence of
A. tuberculosis, diphtheria, pertussis,tetanus,polio,measles,hepatitis A
B. tuberculosis, diphtheria,pertussis,tetanus,polio,measles,hepatitis B
C. tuberculosis,diphtheria,tetanus, pertussis,german measles,polio,hepatitis B
D. tuberculosis, diphtheria,polio,measles,hepatitis A,tetanus,hepatitis B
Recall
16.
Which of the following is included in the extended family?
A. Sister-in-law
C. illegitimate child
B. Step-parents
D. Foster children
Recall
17.
Which of the following Stage Critical Tasks will a Long Term Marriage encounter?
A. Differentiation from family of origin
C. Maintaining couple functioning
B. Renegotiating relationship
D. Renewing marriage commitment
Recall
18.
Which of the following family is much more closely knit?
A. upper class
C. lower class
B. lower class
D. lowest class

Recall

35

19.

A child experience conflict with practices in the home and school regulations is at what
Stage of Family Life Cycle?
A. The Launching family
C. The Newly Married Couple
B. The Family with young children
D. The Family with adolescent

Analysis
20.
Extension of Coverage of NHIP members includes which of the following
A. working mother
B. 22 years high school student
C. 40 years old congenital cerebral palsy patient
D. 63 years old active Police officer
Recall
21.
Which of the following surgical procedures has the highest RVU?
A. Ruptured appendix w/ abscess or generalized peritonitis
B. Cholecystectomy w/ exploration of common duct
C. Dilation and curettage
D. T.A.H.B.S.O
Recall
22.
Which of the following is a case of NHIP Benefits forfeiture
A. Vehicular accident patient confinement in a non-accredited hospitals
B. Patient died within the first 24 hours
C. Patient was transferred to a better equipped hospital before the first 24 hours of
confinement
D. Patient was discharge within the first 24 hours of confinement by an accredited
hospital
Recall
23.
One of the responsibilities of the Occupational Medicine Practitioner is to administer:
A. Periodic examination exclusively done for high risk groups
B. Pre placement examination exclusively done for high risk groups
C. Special examination exclusively done for high risk groups
D. Efficient record keeping exclusively done for high risk groups
Recall
24.
Which of the following is considered an Occupational Disease?
A. Diabetes mellitus
C. Acid peptic disease
B. Hypertension
D. Pneumoconiosis
Recall
25.
Which of the following is an intrinsic factor in occupational disease causation ?
A. social customs
C. organizational set up
B. socio-economic class
D. disease agents
Application
26.
Which branch of statistics are you using when you compute for the average age of a
sample of 100 medical students?
A. descriptive statistics
C. estimation of parameter
B. inferential statistics
D. hypothesis testing
Recall
27.
An advantage of using secondary data is that:
A. data are usually complete.
B. accuracy of data is assured.
C. relevance of data is not usually a problem.
D. data are often available and accessible.
36

Analysis
28.
A City Health Officer was notified of 10 cases of what seemed to be like
Dengue fever. He wanted to determine if this was a beginning epidemic. Where can
he get information on additional cases, if any?
A. registries of diseases
C. hospital records
B. reports of notifiable diseases
D. community clinic records
Recall
29.
An attribute of statistical data which reflects the consistency of data when more than
one observation is done is:
A. sensitivity
C. validity
B. specificity
D. precision
Recall
30.
The ability of a diagnostic test to produce results closest to the true value of the
measurement is called:
A. sensitivity
C. validity
B. predictability
D. precision
Recall
31.
The normal distribution or Gaussian curve is described as having the following
characteristic:
A. mean is zero
C. standard deviation of +/- 1
B. probability of 0.05
D. mean, median and mode are equal
Recall
32.
The area under the normal distribution curve representing observations that lie
between 2 standard deviations from their mean is equal to
A. 68.50%
C. 95.00%
B. 90.5%
D. 99.70%
Recall
33.
For a distribution that is skewed to the left, the mean would be:
A. at the same position as the median
C. to the right of the median
B. to the left of the median
D. equal to the mode
Recall
34.
If the probability that a particular event (A) will occur depends on or is influenced by the
prior occurrence of another event (B), that probability is called
A. a piori probability
C. conditional probability
B. empirical probability
D. complementary probability
Application
35.
Which of the following is a nominal variable?
A. nutritional status
B. outcome of illness

C. serum glucose level


D. body temperature

Recall
36.
The rationale for the use of the Yates correction factor to offset the no expected values
less than 5 is to
A. make the chi-square value smaller
B. make the p-value smaller
C. make the difference between the observed and expected values smaller
D. make the confidence interval smaller

37

Recall
37.
In a 3 x 4 contingency table, the degree of freedom is
A. 2
C. 7
B. 6
D. 12
Recall
38.
At the conventional level of significance, an observed difference between 2 proportions is
considered significant if:
A. p-value is < 0.05
C. chi-square value is >2
B. z-value is > 5
D. confidence interval is > 2SD
Recall
39.
Which of the following is used to show probable relationship between two quantitative
variables?
A. Component bar diagram
C. Bar graph
B. Scatterpoint diagram
D. Pictogram
Recall
40.
Which of the following is an alternative to the pie chart?
A. Bar graph
C. Frequency Polygon
B. Component bar diagram
D. Histogram
Recall
41.
Which of the following is used to portray absolute or relative frequencies, population
rates, or other numerical measurement across the categories of a qualitative or a discrete
quantitative variable?
A. Bar graph
C. Frequency polygon
B. Line Diagram
D. Histogram
Recall
42.
Which type of data presentation is appropriate if the intention is to show detailed
information?
A. Textular presentation
C. Graphical presentation
B. Tabular presentation
D. All of the above
Recall
43.
Which of the following is/are requirement/s for Questionnaire
A. it should have sensitivity, specificity, and positive predictive value
B. respondents are not expected to know all the answer to the questions
C. should be ambiguous
D. fairly offensive
Recall
44.
Secondary data are collected by which of the following methods of data collection?
A. review of literature/documents
C. inquiry
B. survey
D. observation
Recall
45.
Chronologically, which of the following steps in Questionnaire Preparation should be
last?
A. List the variables to be measured
C. Prepare the draft of the question
B. Pretest the questionnaire
D. Make necessary modification
Recall
46.
If death occurred within the jurisdiction of the Philippines but the exact place of its
occurrence cannot be determined, the death shall be registered in the office of the local
civil registrar of
A. place of burial upon the presentation of a certificate
38

B. victims last known address upon the presentation of a certificate


C. birthplace of the victim upon the presentation of a certificate
D. none of the above
Recall
47.
Which of the following should be accomplished if a less than 7 months fetus dies 24
hours
after birth?
A. birth certificate only
B. death certificate only
C. both birth and death certificates
D. both birth and death certificates marked for Statistical Purposes Only
Recall
48.
The process of obtaining a sample where the probability of inclusion of a unit is not
known specified is observed in which of the following sampling methods?
A. Simple Random sampling
C. Stratified Sampling
B. Systematic sampling
D. Quota sampling
Recall
49.
Importance of appropriate determination of sample size include/s:
A. the expense ,time and work is increased
B. the feasibility of the study in terms of availability of the subject is assured
C. the duration of study and estimate of the budget can be extended beyond the
estimates.
D. the precision of the study is diminished
Application
50.
An investigator wants to undertake a study involving the incidence of drug abuse among
Filipinos, which of the following methods is the most appropriate sampling method to be
used?
A. Simple Random Sampling
C. Systematic Sampling
B. Stratified Sampling
D. Multistage Sampling
Recall
51.
Living disease agents that readily die outside the human host require for their successful
transmission requires:
A. an intermediate host
C. a common vehicle
B. a vector
D. the most direct contact
Recall
52.
Herd immunity depends on the following factors
A. degree to which an affected individual is capable of transmitting the infection
B. the length of time during which an individual is infectious
C. size and social behaviour of the community
D. all of the above
Recall
53.
Characteristics of passive immunity
A. it is temporary
B. it usually lasts 6 months in the newborn
C. it protects an individual for life
D. A and B
Recall
54.
It is the first stage in the natural history of disease wherein disease has not yet developed
39

although the groundwork has been laid through the presence of factors that favor its
occurrence
A. Prepathogenesis
C. Early clinical stage
B. Early Pathogenesis
D. Late clinical stage
Recall
55.
The level of prevention that is aimed at intervening before pathological changes have
begun
during the natural history stage of susceptibility
A. Primary prevention
C. Tertiary prevention
B. Secondary prevention
D. none of the above
Recall
56.
A pattern of disease occurrence in a community where the greater proportion of the cases
are
subclinical illustrates the
A. Interference phenomenon
C. Berksonian bias
B. Iceberg phenomenon
D. Pearls fallacy
Recall
57.
Certain diseases that do not confer immunity affect all age groups but the incidence rates
are generally among the very young and the very old because
A. they generally are more exposed to such diseases
B. they generally have lower general resistance
C. they have had no experience with such diseases
D. immunization protects them from acquiring the disease
Recall
58.
Diseases which confers long lasting immunity are highest at
A. younger age groups
C. older age groups
B. middle age groups
D. very young and very old age groups
Recall
59.
The most important personal variable related to disease occurrence is
A. age
C. marital status
B. sex
D. socio-economic status
Recall
60.
Death rates are higher for males than females throughout life and this may be due to the
following
A. sex-linked inheritance
C. differences in hormonal balance
B. environment
D. all of the above
Recall
61.
The M:F ratio for Acute Myocardial Infarction mortality declines by age 40 because
A. females lost the protective hormones
B. females live longer
C. more females than males by this age and after
D. females seek medical care more than the males
Recall
62.
The most direct measure of socio-economic status
A. occupation
C. family income
B. family size
D. area of residence
Recall

40

63.
of

Observed differences in disease frequency can be due to errors of measurement in which


the following?
A. differences in access to medical care
B. differences in precision of diagnosis

C. physiologic differences
D. A and B only

Recall
64.
In which particular situation will you suspect the disease to be a place disease?
A. The disease has a high occurrence in the native inhabitants but not among the
immigrants.
B. The disease has a high occurrence in both the native inhabitants and the
migrants to the place.
C. The disease will continue to have a high occurrence among the offspring of the native
inhabitants who have migrated out of the area.
D. The disease has a low occurrence among the native inhabitants and a high occurrence
among the migrants.
Recall
65.
In the Philippines, generally, variation in mortality rates between the cities and the
provinces
are due to differences in:
A. genetic susceptibility of the populations
B. exposure to the risk factors
C. availability and accessibility of medical care
D. accuracy and completeness of diagnosis and reporting
Analysis
66.
Which of the following can best explain the higher occurrence of breast cancer
among the second generation Chinese, Japanese and Mexican migrants than the
first generation migrants?
A. The second generation migrants lost the protection provided by their genetic make-up.
B. The first generation migrants partly retained their traditional habits and practices.
C. The second generation migrants were exposed to the risk factors for a longer
period
of time.
D. Breast cancer has a long latency period and therefore, would be expressed more in the
second generation migrants.
Recall
67.
High herd resistance to an infectious disease limits the occurrence of the disease
in a community because:
A. It prevents exposure of the population to the disease.
B. It blocks the transmission of the disease.
C. It shortens the transmission time.
D. It reduces the chances of death among the cases.
Recall
68.
Changes in disease frequency usually encompassing several decades are referred to as:
A. Periodic trends
C. cyclic intrinsic variations
B. secular trends
D. cyclic extrinsic fluctuations
Recall
69.
If the line graph of mortality of a particular disease showed an abrupt dip or
decrease, this would most likely be due to:
A. an increase in herd resistance of the population
B. an improvement in diagnostic procedures
C. an improvement in treatment modalities
41

D. altered procedures in assigning causes of death


Recall
70.
Higher patterns of non-infectious diseases during certain seasons are probably due to
A. accumulation of susceptibles
B. change in the climate facilitating multiplication of disease agent
C. change in climate modifying human activities
E. change in disease reporting
Recall
71.
A Case-report is commonly done for
A. statistical and registration purposes
B. legal purposes
C. documenting an interesting medical oddity
D. generating hypothesis
Recall
72.
Which type of study determines/measures both the exposure or factor and the effect or
outcome at the same time?
A. Case-report study
C. Cross-sectional study
B. Case-series study
D. Ecological study
Analysis
73.
Which of the following study is appropriate if an investigator wants to compare the 2 and
5 years survival rates of leukemia if the subjects are randomly given a certain drug X?
A. Case Series studies
C. Case Control studies
B. Cohort studies
D. Experimental studies

Analysis
74.
Which of the following is most appropriate if the objective of the study is to determine if
hypotension increases the chance of fatal outcome in MI patients
A. Case Series studies
C. Case Control studies
B. Cohort studies
D. Experimental studies
Recall
75.
The initial step in the analysis of data in case-control studies is
A. determination of the scale of measurement
B. assessment of the factor
C. computation of the indicators
D. assessment of causality
Recall
76.
The appropriate indicators in Case-control study is
A. the frequency/prevalence of the independent variable in both the cases and
controls
B. the frequency/prevalence of the dependent variable in the exposed and unexposed
groups
C. the measure of risk of occurrence
D. existence of statistical association
Recall
77.
Statistical association found between a factor and disease could be causal if the
association is strong; the strength of association in Case-control studies is measured by
the
A. Relative risk
C. Odds ratio
42

B. Attributable risk

D. Risk ratio

Recall
78.
Which of the following statements is true about cohort studies?
A. They are also called prevalence studies.
B. Subjects are chosen on the basis of whether they have the outcome or not.
C. They begin with identification of a risk group.
D. Actual risk of the outcome cannot be determined using cohort studies.
Recall
79.
Which is an advantage of a concurrent over a non-concurrent cohort study?
A. smaller sample size
C. less biases
B. shorter follow-up period
D. direct measurement of outcome
Recall
80.
A requirement for validity of cohort studies is that:
A. subjects should be representative of target population.
B. exposure variable should be randomly distributed between the study groups.
C. study groups should be similar in all respects except for the exposure.
D. investigator should be blinded to eliminate bias.
Recall
81.
A variable other than the one under investigation which may be associated with
both the factor and the outcome and thus may produce a spurious association is called
a/an:
A. independent variable
C. confounding variable
B. dependent variable
D. misclassification variable
Recall
82.
Which of the following is not a requirement of a descriptive study?
A. Sample representative of target population
B. Controls
C. Similarity of comparison groups
D. All of the above
Recall
83.
Which of the following is TRUE with regards to the subjects of an Experimental study?
A. Sample representative of target population
B. well and diseased persons can included in the sample in a trial of preventive measure
C. diseased persons can be the study population in a curative trial
D. all of the above
Recall
84.
Which of the following is a strength of Experimental study?
A. appropriate for diseases with low or rare occurrence
B. can study multiple effect of a single exposure
C. allows artificial manipulation of the study factors
D. all of the above
Recall
85.
Which of the following statement is TRUE?
A. Blinding aim to eliminate the effects of intervention
B. Placebo administration aim to eliminate the effects of unintended factors
C. Both blinding and Placebo administration aim to eliminate the effects of intervention
D. Both blinding and Placebo administration aim to eliminate the effects of intended
factors
43

Recall
86.
The strongest of all research study is:
A. Uncontrolled Trial
B. Parallel Controlled Trial

C. Cross-Over Trial
D. Quasi-experiment

Recall
87.
In an experimental study, if groups show dissimilarity, adjustments should be done in the:
A. Measurement of baseline variable
C. measurement of outcome variable
B. Application of treatment
D. analysis of data
Recall
88.
Which of the following is most vulnerable to Hawthorne effect?
A. Parallel controlled trial
C. Uncontrolled trial
B. Cross-over trial
D. None of the above
Recall
89.
Which of the following trial should be use to determine the effectiveness of a health
program?
A. Community trial
C. Therapeutic trial
B. Clinical trial
D. Prophylactic trial
Recall
90.
Dengue is currently occurring in epidemic proportion in several areas in the country
because
A. the existing strain of the disease agent has become more virulent
B. a new strain of the disease agent has been introduced
C. of increased opportunities for transmission
D. of inadequacy of health services
Recall
92.
To establish the existence of an outbreak/epidemic, which of the following should be
done?
A. construct the epidemic curve
B. determine the onset of the epidemic
C. compare the present incidence with the normal expectancy
D. compute and compare the attack rate
Recall
93.
Reducing or exhausting the susceptibles in the area to prevent new cases from developing
would be accomplished if
A. they are quarantined
B. they are attacked by the disease
C. the cases are isolated
D. the cases are adequately treated
Recall
94.
If quarantine is necessary to contain an outbreak/epidemic, who of the following should
be
quarantined?
A. Cases
C. contacts
B. Carriers

D. suspects

Recall
95.
Which type of study is generally utilized in investigating outbreak/epidemics?
A. case series
C. cohort
44

B. case-control

D. interventional

Recall
96.
The initial step in planning for data collection is:
A. Determine the research objectives
C. Specify source of data
B. Identify the data to be collected
D. Choose the appropriate method
Recall
97.
Planning for data collection is based on the:
A. research objectives
C. number of subjects
B. data to be collected
D. time for data collection
Recall
98.
The main consideration in the choice of the method of data collection is the:
A. ability of the method to yield measurements or values that is closest to the true
value
B. availability of resources for data collection
C. acceptability of the method to the subjects
D. ability of the method to achieve good coverage of the study population
Recall
99.
The probability that a test will declare positive those who have the disease is:
A. sensitivity
C. positive predictive value
B. specificity
D. positive likelihood ratio
Recall
100. Characteristic of selection of appropriate and adequate subjects includes:
A. A vital stage in any research activity.
B. Begins with determination of suitable reference or target population and ends with
response of the selected persons to the invitation to join the study.
C. It influences the validity of the study.
D. All of the above
B___1. Adopted children are considered members of:
A. Blended family
B. Nuclear family
C. Extended family
D. Consanguineal family
B___2. Community interaction and independence are encouraged in this family type:
A. Random type family
B. Open type family
C. Close type family
D. Corporate family
B___3. This family assessment tool enables the physician to determine family dynamics
and family health problems:
A. Family Genogram
B. APGAR
C. SCREEM
D. Family Circle
A___4. Present day Filipino families have accepted this family phenomenon:
A. Husband no longer monopolizes as the bread winner
B. Institutionalization of the aged member
C. Parenting is restrictive
D. Authoritative parents
A___5. Family Life Cycle provides the physicians the most important information

45

regarding:
A. sequence of stressful events with familys readjustments to maintain health

and viability
B. weak points are identified for the physician to strengthen
C. bad practices for the physician to correct
D. the dominant person in the family is identified for the physician to solely rely on
him/her
C___6. Considered most challenging stage of the family life cycle :
A. Unattached young adult
t
B. Newly married couple
C. Family with young children
D. Family with adolescent

D___7. This stage of family Life Cycle is characterized by the shifting generational roles:
A. Newly married couple
B. Family with adolescent
C. Launching family
D. Family in later years
D___8. A time for positive growth and creative exploration for the entire family
A. Unattached young adult
B. Newly married couple
C. Family with young children
D. Family with adolescent
C__9 . The most important aspect of health promotion among adolescents
A. Screening
B. Immunization
C. Health education
D. Counseling

A__10. Almost 50% of pregnant Filipino women are suffering from::


A. anemia
B. scurvy
C. vitamin deficiency
D. lack of iodine
C__11 Median urinary iodine level that indicates mild IDD
A. < 20 ug/L
B. 20-49 ug/L
C. 50-99 ug/L
D. > 100 ug/L
C__12. Daily vitamin A intake of pregnant women should not exceed
A. 500 iu/day
B. 5,000 iu/day
C. 10,000 iu/day
D. 20,000 iu/day
E. 200,000 iu/day
B__13 Most frequently reported debilitating chronic disorder among the elderly:
A. asthma
B. hypertension
C. diabetes mellitus
D. arthritis

46

C__14 Triage is done during this phase of the Emergency Medical Services system:
A. pre-treatment phase
B. preliminary care phase
C. definitive care phase at the receiving facility
D. recuperation phase

A___15. Which of the following is the real primary target of National Immunization Day?
A. Poliomyelitis
B. Tuberculosis
C. Measles
D. Diptheria, pertussis, tetanus
D___16 The main goal of the countrys CARI program is:
A. Early detection of pneumonia cases
B. Prompt treatment of Pneumonia cases
C. Efficient clinical assessment and management of Pneumonia cases
D. Reduction of Pneumonia deaths
A__ 17. To lessen mortality from pneumonia, the CARI program endorses:
A. early diagnosis of pneumonia cases
B. hospital admission of all patients with pneumonia
C. giving oral antibiotics to all children with cough and colds
D. influenza immunization
C___18. With the emergence of cardiovascular as the leading cause of mortality, DOH
formulated the National Cardiovascular Disease Prevention and Control
Program aimed primarily to:
A. identify high risk individuals
B. treat all identified cases of cardiovascular disease
C. promote healthy lifestyle
D. promote the use of indigenous drugs to treat cases
D__19. Inaccessibility to health care in the Philippines is mostly due to:
A. migration of doctors to other countries
B. lack of health centers and barangay health stations
C. lack of nurses
D. most hospitals, health care facilities and doctors are urban based
B___20. Increased prevalence rate is seen in:
A. increasing population
B. increased survival
C. increased incidence
D. epidemics
B___21. A stage in the Family Life Cycle where parents and grandparents are
encountering crisis as well:
A. Launching
B. Family in later years
C. Family with adolescents
D. Family with young children
A.__22. An example of primary prevention is:
A. Measles immunization
B. self examination of the breast for tumors
C. smoking cessation after a heart attack
D. cervical cytology screening
B__ 24. Births in the Philippines are registered in the municipality or city :
A. where the infants parents live
B. where the infant was born
C. of the parents choice
D. listed as the familys permanent residence

47


C __25. The cause of death coded for the purpose of tabulating mortality statistics is:
A. Immediate cause of death
B. intermediate cause of death
C. underlying cause of death
D. antecedent cause of death
B__26 . The registration of death in the Philippines as specified by law must be done
not later than:
A. 24 hours after death
B. 48 hours after death
C. 2 weeks after death
D. 30 days after death
C__27.
A.
B.
C.
D.

Deaths in the Philippines are registered in the municipality or city::


where the deceased was born
of the persons usual residence
where the person died
listed as the persons permanent address

A__28 Accumulation of this metal in organs and tissues of humans has been linked to
brain damage:
A. mercury
B. aluminum
C. lead
D. silver
B__ 29 This CFC offender is ozone safe and does not contain chlorine:
A. HCFC (hydrochlorofluorocarbons)
B. HFCs (hydrofluorocarbons)
C. Hydrocarbons
D. Chlorofluorocarbons

B__ .30. Method of food preservation which prevents multiplication of bacteria without
destroying them:
A. preserving
B. refrigeration
C. fermentation
D. pickling
C__31 . Most important test to determine if the water is potentially dangerous:
A. physical examination
B. biological examination
C. bacteriological examination
D. chemical examination
B___32. The coliform count from a well is 35/100 cc. The treatment required is:
A. Rejection of sources
B. Standard treatment
C. Double treatment
D. Triple treatment
A___33. Milk is best preserved by this method:
A. Sterilization
B. Boiling
C. Freezing
D. Adding sodium benzoate

48

C___34. Of the following devices for excreta disposal, the most dangerous in
contaminating ground water is:
A. Chemical privy
B. Pail system
C. Bored hole latrine
D. Septic tank
B___35 The concentration of which of the following greenhouse gases is affected by
deforestation
A. Water vapor
Chlorofluorocarbons
C. Methane
D. Nitrous oxide

B.

B___36. Formation of sticky precipitates of soap with water indicates that the water is:
A. Polluted
B. Hard
C. Contaminated
D. Soft
D___37. Increase in which of the following in drinking water is associated with
Alzheimers disease
A. Antimony
B. Zinc
C. Silver
D. Aluminum
B___38. Dissolved gases and odors in water are removed by:
A. Filtration
C. Sedimentation
B. Aeration
D. Coagulation
D___39. Chlorination of water is directed towards:
A. Bad taste
C. Turbidity
B. Odor
D. Bacteria
B___ 40. The recommendedl refuse disposal method for hospital is:
A. Dumping
B. Incineration
C. Recycling
D. Composting
A___41. The cheapest and most frequently used refuse disposal in the community
but can result to water and air pollution is:
A. Dumping
B. Sanitary landfill
C. Incineration]
D. Reduction and salvage
C__42.
A.
B.
C.
D.

The first step in an epidemiologic/outbreak investigation;


Formulate the hypothesis
Characterize the epidemic
Determine the existence of an epidemic
Implement control methods

B__43. The ultimate goal of an epidemiologic investigation:


A. To request for international aid
B. Institute curative, preventive and control measures to avoid more cases
C. Identify administrative and operational problems of a health agency
D. To determine discrepancies or gaps in policy implementation
A___44. Primary Health Care Concept is:
A. meant for people to attain health by being responsible for it

49

B. the provision of all health care services at affordable prices


C. meant to give accessible, affordable and available primary, secondary and tertiary
care
D. a level of care
B__45. Direction of personnel and techniques in operation as a component of public
health administration is:
A. organization
B. management
C. staffing
D. performance evaluation
A__46 . Community Diagnosis is usually regarded as:
A. situational analysis
B. analysis of needs
C. health survey
D. health program planning
C__47. In the model of a Health Care Delivery System, input refers to:
A. health activities
B. health products
C. health resources
D. feedback
A__48. Regulation of importation, manufacture, marketing, and consumer utilization of
all drugs refer to:
A. quality control
B. rational use of drugs
C. development of self-reliance in the local pharmaceutical industry
D. tailored or targeted procurement of drugs by the government
D__49. Control as a component of a health system model is exemplified by:
A. immunization programs
B. number of fully immunized children
C. vaccines and reagents
D. immunization protocol
C___50. In community health planning, the BEST people to identify and prioritize
problems are the:
A. Community doctors
D. Social worker
B. Volunteer health workers
E. Nurses/midwives
C. Community people themselves
C__51. Recognition of each ones roles and functions in a multidisciplinary team is
A. Comprehensive
B. Continuous
C. Teamwork
D. Coordinated
B__52. In a planning cycle, analysis of why programs and projects fail is:
A. Situational analysis
B. Evaluation
C. Impact analysis
D. planning
A__53. With the implementation of the Local Government Code, municipal and city
health services are placed under the supervision of the:
A. City and municipal mayors
B. Doctors of district hospitals
B. DOH
C. PGH

50

D__54. Which of the following will make a country a priority for international aid?
A. IMR of 5/1000 livebirths
B. IMR of 15/1000 livebirths
C. IMR of 20/1000 livebirths
D. IMR of 50/1000 livebirths
B__55. The ideal denominator for Maternal Mortality Rate but cannot be used :
A. Total livebirths
B. Number of pregnant women
C. Total births
D. Midyear population
A__56. Most commonly used index of fertility:
A. Crude Birth Rate
B. General Fertility Rate
C. Age Specific Birth Rate
D. Total Fertility Rate
C__57. Regarding exposure and outcome a relative risk of 15 for a certain disease
means:
A. Non causal association
B. Beneficial association
C. Harmful association
D. Definite causal association

B__58. Swaroops index in an indicator of the following:


A.
B.
C.
D.

medical , social and economic condition of the area where it is applied


a better chance for longer survival given to a population
relative importance of different diseases as cause of death
the killing power of a disease

B__59. The first line of defense against occupational hazard:


A. personal protective equipment
B. engineering control
C. ergonomics
D. good nutrition

A__60. For asymptomatic and normotensive elderly, blood pressure monitoring


should
A.
B.
C.
D.

be:
Every month
Every 6 months
Every year
Whenever convenient

A__61. For high risk elderly women Paps smear should be done:
A.
B.
C.
D.

Annually
Every 2 years
Every 3 years
Every 5 years

C__62. The earliest reliable predictor of pneumonia in the CARI Program is:
A. cyanosis
B. chest indrawing
C. tachypnea
D. lethargy
B__63. In the CARI Program a child with productive cough must be given:
A. Antitussives

51

B. A lot of fluids
C. Expoectorants
D. Antibiotics
C__64. In the CARI Program the diagnosis for a 6-month old child with RR = 60/min,
Chest , indrawing, intercostal and subcostal retraction but without cyanosis
A. No pneumonia
B. Pneumonia
C. Severe pneumonia
D. Very severe pneumonia
A__ 65. The first line of antibiotic regimen under the CARI Program:
A. oral Co-trimoxazole
B. procaine penicillin
C. chloramphenicol
D. gentamicin
B__66 So that an epidemic of chicken pox will not occur the best control measure is:

a. quarantine of cases
b. isolation of cases
c. control of animal reservoir
d. surveillance of cases
D__67.
A.
B.
C.
D.

Success in ORT of a child with diarrhea at home is achieved by:


the doctor explaining to the mother/caretaker how it is done
having posters on the clinic wall showing how ORT is done
a nurse or health worker demonstrating ORT
the mother/caretaker practicing giving ORT guided by a health worker

B__68. The main thrust in the control of diarrhea by the DOH:

A. environmental sanitation
B. oral rehydration therapy with ORESOL
C. public education
D. immunization
A__69. A very early sign of dehydration in a child with diarrhea:
A. thirst
B. sunken eyes
C. increased pulse and breathing rates

D. somnolence
D__70. Primary reason for inclusion of glucose in oral rehydration solution:
A. as a source of calorie
B. for palatability
C. to increase the nutritive value
D. to help in the absorption of sodium
B__71. A. proportionate Mortality indicator:
A. Age-sex specific death rate
B. Swaroops index
C. Infant Mortality Rate
D. Neonatal Mortality Rate

B__72. To establish ways to improve the overall health status of


communities the best
experimental approach to use is:
A. Clinical trial
B. Community trial

52

C. Therapeutic trial
D. Intervention trial

B__73. The most accurate relative risk is the one established in:
A. case-control study
B. cohort prospective
C. cohort retrospective
D. cross-sectional study
E. all of the above

B__74. To determine the prevalence of a disease this study design is used


A. Descriptive
B. Cross sectional
C. Case control

D. cohort
E. experimental

C__75. A cross-sectional study is not applicable for this purpose:


A. determine the magnitude of diseases
B. establish baseline data
C. study causes of disease
D. study rare diseases
D__76. Controls in a case-control study are those:
A. who will constitute the comparison group

B. who will not develop the outcome


C. who will not be given the experimental drug
D. who do not have the illness being studied

C__78. The most important thing to consider in the selection of cases in a case-control
study:
A.
B.
C.
D.

That the cases will definitely show positive exposure


That the cases can be interviewed regarding past experiences
That the cases satisfy the specific diagnostic criteria
That the cases are typical of the patients to be studied

C__79. Not true of cohort studies:


A. Investigators still have to wait for the outcome
B. Controls are those without exposure or risk factor
C. Intervention is given to the exposure group
D. Study moves forward with time
C__80. Statement/s that describes an experimental study:
A. The investigator can only observe occurrence of disease in people under
natural setting
B. Contrast in outcome between study groups are observed, not created in a
controlled way
C. The investigator controls conditions under which the study is done
D. The investigator observes disease occurrence in people who are grouped on the
basis of experience or exposure.

53

D__81. The study design where the investigator has control over the conditions of the
study:
A.
B.
C.
D.

cross-sectional
case-control
cohort
experimental

D__ 82.Experimental studies are concerned with


A. Investigating hypothesis regarding causal association
B. Measuring effects on animals of introducing in a controlled way new cures for
diseases
C. Observing distribution of diseases in population
D. Testing hypothesis through direct control over assignment of subjects to study
groups

D__ 83. Intervention trial is exemplified by:


A.
B.
C.
D.

Simple mastectomy for breast cancer


BCG vaccination for prevention of TB
Isoniazid for TB prophylaxis
Antihypertensive drugs to minimize stroke

B__ 84. The probability sampling method that assures adequate representation of
minority sub-groups
A. Systematic sampling
B. Stratified sampling
C. Cluster sampling
D. Simple random sampling
:
C__85. Management of infants with diarrhea under the CDD Program
A. Rest the stomach by fasting for a day
B. Shift from breastfeeding to lactose free formula
C. Add vegetable oil to food
D. Start co-trimoxazole after 6 episodes of diarrhea
D__86.
A.
B.
C.
D.

The most effective method to control TB in the Philippines is:


Isolation of TB patients
Mass chemoprophylaxis with INH
Mass chest x-ray and treatment of cases
Mass BCG vaccination

B__ 87.The most common complication associated with BCG vaccination is:
convulsion
suppurative lymphadenitis
encephalitis
reactivation of TB
B__ 88.The management for an infant who developed a 10 mm red, tender swelling at

54

A.
B.
C.
D.

the BCG site is:


application of topical antibiotic on the swelling
assurance of the mother that no treatment is needed
application of pulverized INH on the swelling
prophylactic INH at 10 mg/kg/day

C__89. Occurrence of a group of illness of similar nature clearly in excess of normal


expectancy;
A. Holoendemic
B. Hyperendemic
C. Epidemic
D. Pandemic
C__90 The rate used as an indicator of an outbreak or an epidemic:
A. Morbidity rate
B. Case fatality rate
C. Incidence rate
D. Prevalence rate
A__91. A graphical presentation of data most useful in locating cases during an
epidemic:
A. Spot map
B. Pictograph
C. Scatter diagram
D. Histogram

C__92. Risk considered due to exposure:


A.
B.
C.
D.

relative
absolute
attributable
odds ratio

A__93. The one that establishes cause-effect relationship is:


A. relative risk
B. absolute risk
C. attributable risk
D. odds ratio
B__94. Randomized control trial means:
A. randomization was used and there is a control group
B. randomization was used, there is a control group and conditions are controlled by
the investigator
C. randomization was used, there is a control group and the study is blinded
D. randomization was used, there is a control group, the study is double blinded
D__95. Cross-over trial means:
A. with separate comparison groups
B. with separate control groups
C. no control group
D. subject and control are the same persons
D__96. To encourage community participation, the local doctor should do the following:
A. invite people from civic organizations
B. refer to specialist if the need arises
C. encourage sponsorship of programs
D. listen when people relate their problems
B__97. In the Philippines, the age range most infected with HIV is:
A. 15 24 years old
B. 25 39 years old
C. 35 - 44 years old
D. 45 55 years old

55

B__98. The most important mission of the Department of Health


A. Promote healthy lifestyle
B. Ensure accessibility and quality of health care
C. Reduce morbidity and mortality
D. Improve the general health status of the people
C__99. Which diagnostic test is constantly reviewed in the National TB program?
A. Bronchoscopy
B. Chest x-ray
C. Sputum culture
D. Tuberculin skin test
C__100. If a pregnant woman received 3 doses of Tetanus Toxoid she will have an
immunity against tetanus for how many years?
A. 3 years
B. 5 years
C. 10 years
D. 15 years
C__101. The mother of a 5-month old infant claimed that her baby had received
complete immunization for his age. One can assume that the baby has already
:received:
A. BCG and measles vaccine
B. BCG, 3 doses of OPV and 1 measles vaccine
C. BCG and 2 doses of DPT vaccine
D. BCG, 3 doses of OPV, DPT, Hepatitis B vaccine, 1 measle4s vaccine

1. Primary health care as defined in Alma Ata is referred to as:


A. first contact between a physician and the patient
B. strategy utilizing participatory approach in the provision of basic health
services to the people.
C. training program on first aid for the people in the communities
D. a program of health activities carried out by health professionals
2. A characteristics feature of Primary Health Care include one of the following:
A. community participation
B. infrastructure development and modernization
C. emphasis on curative medical care
D. training of health professionals
3. People empowerment in primary health care is brought about through this means:
A. volunteerism among the target population
B. hiring and employment of community health workers
C. community organization process
D. endorsement from church and political community leaders
4. This type of health program allows maximum room for community participation and
empowerment:
A. hospital based program
B. community oriented health program
C. community managed health program
D. community-based health program
5. The basic foundation of the community based and managed health program is:
A. team approach
B. health human rights
C. equality and justice
D. community organization

56

6. The main case finding strategy advocated in the National Tuberculosis Program is:
A. chest radiography
B. sputum microscopy
C. DOTS
D. Mantoux test
7. Based on the national Malaria control program, the currently recommended principal
measure directed against the mosquito vectors in Malaria is:
A. biologic vector control
B. residual spraying with DDT
C. use of chemical impregnated mosquito nets
D. sterilization of male mosquitoes
8. The Integrated Management of Childhood Illnesses (IMCI) requires the delivery of the
following service(s) for a child who visit a physicians clinic for any health related
problems:
A. routine immunization
B. breastfeeding education for the mother
C. proper and adequate nutrition counseling
D. all of the above
9. The most cost effective preventive measure for the control of cardiovascular diseases:
A. CVD screening using ECG
B. environmental cleanliness
C. primordial prevention of CVD risk factor
D. genetic counseling
10. This is an essential health service that is available, accessible and affordable by the community:
A. social care
B. preventive medicine
C. medical care
D. primary health care
11.
A.
B.
C.
D.

The following level of health care does NOT belong to primary level:
teacher health guardians
emergency hospital
rural health unit
rural health midwife

12. Which of the following measures has the greatest impact on community health?
A. provision of social services
B. provision of potable water supply
C. immunization
D. medical care of the sick
13. The health status of a community may be measured or expressed in terms of:
A. availability and utilization of health care services
B. availability of public utilities
C. its statistical indices of morbidity and mortality
D. all of the above
14. Community planning should start with:
A. formation of objectives
B. identification of roles and their relationships
C. educational analysis
D. determination of resources
15. The local health department is usually associated with this type of health care planning:
A. morbidity planning
B. population-based planning
C. institution-based planning
D. financial planning

57

16. The prime yardstick of child health level in the community is


A. infant and child mortality rate
B. number of live births
C. number of preschool children
D. number of health centers
17. Growth monitoring is important in a primary health care program because it
A. tells us how many children are at risk of being malnourished
B. affords an occasion for maternal education
C. tells us how many children are undernourished in the community
D. improves the nutritional status of children in the community
18. Which factor determines the highest use of health care?
A. sex
B. education
C. age
D. income
19. The lowest operating unit of the DOH is the:
A. emergency hospital
B. rural health unit
C. barangay health station
D. district health office
20. Public health centers do NOT need constant supplies of:
A. antidiarrheals
B. analgesics
C. antipyretics
D. antibiotics
21. A primary health care team is composed of:
A. Office of the Secretary of Health
B. Provincial Health Officer and staff
C. Municipal Health Unit and staff
D. Interdisciplinary composition of community people working to gather under professional
supervision
22. The principal and unique strategy of the PHC is:
A. empowerment of the people
B. holistic health care delivery
C. teamwork among health personnel
D. socialization of health care
23. Which of the following is NOT included in the essential health service of Primary Health Care?
A. immunization against the major infectious disease
B. provision of free drugs
C. provision of safe water and basic sanitation
D. appropriate treatment of all existing diseases
24. Medicare benefits include:
A. psychiatry
B. surgery
C. optometry
D. diagnostic tests
25. Under Medicare and most hospitals, the anesthesiologist's fee should NOT exceed _____% of the allowable
surgeons' fee:
A. 10%
B. 30%
C. 40%
D. 20%
26. The 1995 National Health Insurance Program covers the:
A. government employees only
B. indigent patients only
C. all Filipino citizens
D. unemployed citizens

58

27. BCG vaccine is given according to the Expanded Program on Immunization of the Philippines, at what age?
A. 3 months
B. 6 months
C. at birth
D. 9 months
28. Control TB infectivity is best achieved by:
A. isolation
B. personal hygiene and cleanliness
C. prompt specific drug treatment
D. education
29. The Department of Health currently recommends the most effective mode of anti-TB treatment by:
A. two-week monitoring
B. directly observed therapy
C. home visits
D. health center based follow-up
30. During a diarrhea bout, breastfeeding of this infant should be
A. stopped and substitute with a lactose-free formula
B. temporarily discontinued
C. given more often
D. limited
31. A program in the control of communicable diarrheal diseases by the Department of Health:
A. hydrotherapy
B. oresol therapy
C. physiotherapy
D. use of intravenous fluids
32. Screening of children has been proven cost-effective for all of the following, EXCEPT:
A. hypertension
B. iron-deficiency anemia
C. vision impairment
D. tuberculosis
33. The Civil registry Law includes the registration of
A. Births and marriages
B. births, sickness and marriages
C. births, deaths, marriages
D. births, deaths, sicknesses, marriages
34. A 28 day old infant is classified as:
A. neonatal mortality
B. fetal neonatal mortality
C. post-neonatal mortality
D. perinatal mortality
35. The denominator in getting the sex ratio in a population is
A. number of females
B. number of fertile females
C. number of fertile males
D. number of males
36. An investigator reported three cases of carcinoma of the lungs among workers in a cement plant. This type of
study is best described as
A. Cohort
B. Case series
C. Case report
D. Case control .
37. The major activities undertaken in the maternal and child health program does NOT include
A. Immunization of pregnant mothers against measles
B. proper pneumonia prevention
C. promotion of prenatal care
D. monitoring of growth and health status of infants and children

59

38. One of the major determinants of the degree to which chance affects the findings in any particular study is
A. statistical testing
B. sample size
C. randomization
D. bias
39. Hepatitis B spread is NOT known to occur in
A. eating food
B. sexual contacts
C. household contact
D. mother to child transmission in perinatal period
40. What kind of epidemiologic study should be done to determine the cause of an unusual infection?
A. experimental studies
B. analytic study
C. descriptive study
D. none of these
41. One of the following is NOT descriptive of a young population
A. population pyramid with a wide base
B. median age of 20-24 years
C. crude birth rate greater than 30/1000
D. dependency ratio of 1:1
42. Specific health protection can be carried out by
A. Immunization
B. chemotherapy
C. Health education
D. A and B only
43. The Expanded Program on Immunization is concerned with this particular age group:
A. Infants below 1 yr, school entrance, and pregnant women
B. Infants below 6 mos, pre-schoolers, and pregnant women
C. Neonates, pre-schoolers, and post-partum women
D. Infants of any age, school children, and pregnant women
44. The recommended age for the start of DPT-OPV immunization under the government EPI is
A. 8 weeks
B. 10 weeks
C. 6 weeks
D. 12 weeks
45. An example of artificial active immunity is one that is derived from
A. giving of gamma globulin
B. vaccination
C. actual illness
D. transplacental passage of antibodies
46. Controlled studies mean
A. An experiment is being done
B. There is a comparison group
C. the characteristics of the participants are all the same
D. a group is not to be given the experimental procedure
47. A study was done to prove that co-trimoxazole is more effective than other drugs in treating respiratory tract
infections in ambulatory patients in a community. The suitable control group will receive
A. Another drug for respiratory infections
B. The standard drug for treating respiratory infections
C. another experimental drug for respiratory infection
D. a placebo
48. A comparison or control groups is NOT necessary in
A. Prospective studies
B. Retrospective studies
C. experimental studies
D. descriptive studies

60

49. Results are generalized to a comparable population


A. Internal validity
B. External validity
C. bias
D. confounders
50. Double blind determination of the exposure and outcome is for the purpose of
A. Reducing the effects of losses to follow up
B. Reducing the effects of sampling variation
C. achieving comparability of cases and controls
D. avoiding observer and subject bias
51.

Describes the distribution of cases by the variables of person, time, and place in order to study and explain
acute outbreaks of disease and to develop hypothesis about disease transmission:
A. Case control studies
B. Analytic studies
C. descriptive studies
D. Experimental studies

52.

Descriptive studies do
A. Draw inferences about cause and effect
B. Predict outcome
C. not have a sample size estimation
D. not require a hypothesis

53.

The possibility of cause-effect relationship is difficult to assess in


A. Experimental
B. Cross-sectional
C. cohort
D. case-control

54.

A study comparing the prevalence of hepatitis B infection among government and private hospitals falls
under
A. Descriptive study
B. Cohort study
C. case-control
D. cross-sectional

55.

A well-defined group of people who have had a common experience or exposure who are then followed up
for the incidence of new diseases or events is a
A. Variable
B. Sample
C. Case
D. Cohort

56.

Odds ratio is the measure of analysis commonly used in


A. Case report
B. Case-control studies
C. case series
D. cohort studies

57.

A study is planned to determine if there are more users of oral contraceptives among patients with
thrombophlebitis than among well persons. To achieve the objective, which of the following type of studies
is most appropriate?
A. Clinical
B. Descriptive
C. concurrent cohort
D. case control

58.

In a cohort study of the association of leukemia with x-ray exposure, the relative risk was found to be 3.
What does this mean?
A. Those with x-ray exposure have threefold probability of developing leukemia
B. X-ray exposure protects the person from developing leukemia
C. there is no association between leukemia and x-ray exposure
D. none of the above

61

59.

Which of the following study designs most appropriately illustrates this case history: A total of 200 newly
diagnosed patients with endometrial cancer are allocated to treatment with either surgical removal alone or
surgical removal plus radiation treatment
A. Clinical trial
B. Case-control study
C. case series report
D. cohort study

60.

The observational study in which subjects are sampled based on the presence or absence of a risk factor of
interest. These subjects are followed over time for the development of a disease outcome
A. Case-control study
B. Ecologic study
C. cohort study
D. cross-sectional study

61.

In qualitative data, the appropriate measure of central tendency is the


A. median
B. average
C. mean
D. mode

62.

"In a frequency distribution, the value that divides the observed values into two parts is the
A. median and mode
B. median
C. mean
D. mode

63.

If there is an inherent order or progression among categories, they are said to be measure by:
A. numerical scale
B. nominal scale
C. discrete scale
D. ordinal scale

64.

To represent the trend over time in crude birth rate and crude death rate in a given province, the best graph to
use is a:
A. bar
B. histogram
C. line diagram
D. frequency polygon

65.

The rate that measures the probability of a person dying in a year regardless of age, sex, race, etc is:
A. case fatality rate
B. neonatal mortality rate
C. crude death rate
D. specific death rate

66.

The following indices use the midyear population as the denominator, EXCEPT:
A. crude birth rate
B. measles incidence rate
C. crude death rate
D. proportionate mortality rate

72. The following is an indirect method of nutritional assessment:


A. Weight for height
B. Percentage of low birth weight babies
C. Height for age

D. Measles mortality rate


73. The number of live births reported during a given time interval divided by the estimated number of women in the
15-44 year old group at mid-interval is:
A. crude birth rate
B. incidence rate
C. prevalence rate
D. fertility rate
74. This is the most commonly used index of fertility:

62

A.
B.
C.
D.
75.

crude death rate


infant mortality rate
maternal mortality rate
crude birth rate

Turbidity of a given water is due to the:


a. radioactive contaminant in water
b. total mineral content
c. substances in solution
d. impurities in suspension

76. Hardness of water is primarily due to:


a. calcium and magnesium
b. nitrogen content
c. iron content
d. sulphur content
77. Laws and regulations on Food Sanitation are implemented to protect the public against fraud and deceit. Some
of the restraints are the following, EXCEPT:
a. misbranding
b. sale of damage food
c. food adulteration
d. adequate food storage
78. Which of the following is the primary objective of food sanitation?
a. to ensure the consumption of safe and wholesome food
b. to prevent the sale of food which is inferior in value and quality
c. to cut down spoilage and wastage of food
d. none of the above
79. Excreta disposal is considered to be sanitary if
a. there is minimal fecal handling
b. it prevents soil and water contamination
c. it is accessible and acceptable contamination
d. it prevents both soil water contamination and there is minimum fecal handling
80.One of the following is not a domestic sewage:
a. sewage coming from industrial operations
b. sewage coming from commercial establishments
c. sewage coming from households
d. sewage coming from offices
81.Sanitary methods of excreta disposal includes the following, EXCEPT:
a. bored hole latrine
b. septic tank
c. aqua privy
d. pour flush latrine
82. The most practical and sanitary way to excrete disposal in the rural areas is:
a. overhung latrine
b. water-sealed toilet
c. chemical toilet
d. septic toilet
83.A gas produced by the biodegradation of waste is:
a. Methane
b. oxygen
c. carbon monoxide
d. carbon dioxide
84.In the most part of the world, vector control is largely through:
a. genetic manipulation of vectors
b. modifying the environment of vectors
c. use of chemical pesticides
d. extensive public health education
85.The probability that a person has the disease given a positive test result is:

63

a.
b.
c.
d.

negative predictive value of the test


sensitivity of the test
specificity of the test
positive predictive value of the test

86. A physician wants to determine the prevalence of parasitism among school children in an
urban area. There is great variability in the economic status in that locality. On one hand, most
children in private schools come from affluent families, and on the other hand, most children
from public schools come from poor families. The most appropriate type of sampling
methodology to use in this situation is:
A. simple random sampling
C. stratified random sampling
B. systematic sampling
D. multi-stage sampling
87. Philhealth benefits vary according to the following except:
A. Relative unit value of surgery
B. Actual cost of care

C. Type of hospital
D. Type of physician

88. One can be reasonably sure that the hypothesized cause preceded the occurrence of the
disease and that disease status did not differentially influence the selection of subjects by study
factor level in a:
A. Cross-sectional study
C. Case-control using prevalent cases
B. Cohort study
D. Experimental study design
89. The following method of data analysis is most relevant for the cohort study design:
A. Prevalence odds ratio
B. Prevalence rate
C. Odds ratio
D. Relative risk
90. This is a sampling method wherein every subject falling within a fixed sampling interval
is obtained from a population N which is arranged in some definite way:
A. Simple random sampling

B. Systematic sampling
C. Stratified sampling
D. Cluster sampling
91. When a researcher wants to test for the association between two categorical variables, the
appropriate statistic to use is:

a. Chi square
b. T test
c. Analysis of variance
d. Correlation coefficient
93. The rate that measures the probability of a person dying in a year regardless of age, sex,
race, etc is:
A. case fatality rate
B. neonatal mortality rate

C. crude death rate


D. specific death rate
94. Which method of collecting data is devoid of observer error and bias?
A. Interview

B. Self-administered questionnaire
C. Observation
D. Review of documents
64

95. Used to describe the relative importance of different fatal disease in the population of
different age, sex, occupation etc.

a. specific death rate


b. attack rate
c. cause of death rate
d. proportionate mortality rate
96. The peculiar feature of a cross sectional study that distinguishes it from other study
designs is:
A. Information on both the independent and dependent variables are collected
simultaneously at the time of the survey
B. Information on the independent variable is collected first, after which the subjects
are followed up later for assessment of the dependent variable
C. Information on the dependent variable is obtained, after which assessment of the
independent variable is made
D. Information on both the independent and dependent variables are obtained by doing
either queries or actual observation
97. The effect of an inadequate sample size for a study is:
A. Subjects may not be a random representative of the base population
B. There may not be enough subjects that will develop the outcome, in order to detect a
difference
C. The researcher will not be able to apply the appropriate statistical test for the kind of
data available
D. The researcher might find an association between independent and dependent
variables, even if there actually is none
98. The group most vulnerable to nutritional deficiencies:
a. Pregnant women
b. Working group
c. Adolescents
d. Non-lactating mothers
99. The most sanitary source of water is:
a. Groundwater

b. Rainwater
c. Surface water
d. Brackish water
100. The primary disinfectant in water treatment:
a. Fluorine
b. Chlorine
c. Aluminum sulfate
d. Ferric sulfate
SOURCES:
Beaglehole, Bonita, Kjellstrom; BASIC EPIDEMIOLOGY STUDENTS TEXT, World
Health Organization, July 1990
Clark, D., MacMahon, B: PREVENTIVE AND COMMUNITY MEDICINE, 2ND ed 1981
DOH Health Programs Manual
Gupta, M.; Mahajan, B: TEXTBOOK OF PREVENTIVE AND SOCIAL MEDICINE, 3rd ed
2003
65

Jekel, Elmore & Katz; EPIDEMIOLOGY BIOSTATISTICS AND PREVENTIVE


MEDICINE, 1996
Kuzma, Bohnenblust; BASIC STATISTICS FOR THE HEALTH SCIENCES 4th ed.,
McGraw-Hill 2001
Last, J; Wallace, R; PUBLIC HEALTH AND PREVENTIVE MEDICINE, 13th ed, 1992
Philhealth Manual
Sanchez, Morelos, Baltazar, Peralta; RESEARCH METHODS IN HEALTH AND
MEDICINE, Vol. 1 Planning Research (3rd ed.) Philippine Council for Health Research and
Development 1996
Jmn/uermpcmexam questions for apmc may 4 2007
1. This is a branch of medicine that deals with provision of comprehensive health care to
people regardless of age or sex:
A. Public Health
B. Community Medicine
C. Preventive Medicine
D. Family Medicine
Answer: D Proceedings of the Orientation Course in Family Medicine p. 8
3. This is the branch of medicine that deals with the study of the causes, distribution, and control
of disease in populations:
A. Family Medicine
B. Preventive Medicine
C. Epidemiology
D. Public Health
Answer: C Epidemiology by Leon Gordis p. 2
4. Which of the following is an example of chemoprophylaxis for newborns?
A. Varicella vaccine
B. Erythromycin ointment
C. Folic acid
D. BCG
Answer: B Family Medicine Ambulatory Care and Prevention by Mengel et. al p. 736
5. Dr. Hook wants all of his children and their respective families to live with him and his wife
until the time of his death. This is an example of what family structure?
A. Nuclear
B. Extended
C C. Single Parent
D D. Blended
Answer: B Proceedings of the orientation course in Family Medicine pp. 14-15
6. The stepfather and half-brother of Keanna lives with her and her mother. This is an example of
what family structure?
A. Nuclear

66

B. Extended
C. Single Parent
D. Blended
Answer: D - Proceedings of the orientation course in Family Medicine pp. 14-15
7. The family life cycle starts with this stage:
A. Newly Married Couple
B. Family With Young Children
C. Unattached Young Adult
D. Family With Adolescents
Answer C: Proceedings of the orientation course in Family Medicine p. 18
8. In the family APGAR, resolve is elicited by
A. asking if the family supports the patient in his desire for progress
B. looking into patient's desire to openly communicate problems with the family
C. Asking if the patient feels loved by family members
D. inquiring if the patient is given adequate time by the members of the family
Answer D: Proceedings of the orientation course in Family Medicine p. 41
9. Which of the following does not describe the CURRENT state of the Philippine health
situation?
A. Double burden of disease
B. Continuing threat from emerging and resurgent diseases
C. Highly centralized health care system
D. Large variations in health status across population and socioeconomic groups
Answer C: Filipino Report Card on Health, Social Weather Stations Survey 2000
11. A score of 8 on the family APGAR indicates:
A. A highly functional family
B. A moderately dysfunctional family
C. A severely dysfunctional family
D. A large, extended family structure
Answer A - : Proceedings of the orientation course in Family Medicine p. 41
13. Which of the following parameters are evaluated in SCREEM?
A. Entrepreneurial
B. Economic
C. Resolve
D. Mental
Answer B - Proceedings of the orientation course in Family Medicine p. 48
14. A dashed line (-------) that connects a male and female couple in the family genogram
denotes
A. a married couple
B. a legally separated couple
C. a consanguinous marriage
D. live-in partners
Answer D - Proceedings of the orientation course in Family Medicine p. 46

67

16. You should use an 'arrow' symbol in your patient's genogram to signify:
A. The index patient
B. The head of the family
C. Death in the family
D. Provider of the family
Answer A - Proceedings of the orientation course in Family Medicine p. 36
17. Stage in the illness trajectory experienced by the patient and family members prior to contact
with medical care providers
A. Onset of illness
B
B. Impact Phase
C C. Major therapeutic efforts
D D. Early adjustment to outcomes
Answer A - Proceedings of the orientation course in Family Medicine p. 29
18. The most catastrophic form of Cerebrovascular Accident
A. Lacunar
B. Thromoboembolic
C. Hemorrhagic
D. Transient ischemic attack
Answer C Harrison Principles of Internal Medicine 14th ed. P. 2342
19. Chronic bronchitis is a disorder in which productive cough is present in most days
A. for at least 2 months of the year, for more than 3 consecutive years
B. for at least 3 months of the year, for more than 2 consecutive years
C. for at least 2 months of the year, for more than 2 consecutive years
D. for at least 3 months of the year, for more than 3 consecutive years
Answer B - Harrison Principles of Internal Medicine 14th ed p. 1451
20. It measures what is supposed to be measured refers to:
A. Specificity
C. Validity
B. Sensitivity
D. Reliability
Answer D. Epidemiology by Gordis p. 74
21. In Epidemiology, it is defined as the proportion of the true negatives correctly identified by
the test.
A. Specificity
B. Sensitivity
C. Validity
D. Reliability
Answer A. Epidemiology by Gordis p. 59
22. It is one with potentially an infinite number of possible values in any interval.
A. Continuous variable
B. Bulbous variable
C. Categorical variable
D. Discrete variable
Answer A Foundations of statistical analysis for the Health Sciences by Mendoza et. al p. 5
23. Another name for ranking scale, it has an implicit order relationship.
A. Nominal
68

B. Ordinal
C. Interval
D. Ratio Scales
Answer B - Foundations of statistical analysis for the Health Sciences by Mendoza et. al p. 5
24. The most frequently occurring value in a series of observations:
A. Frequency distribution
B. Median
C. Weighted mean
D. Mode
Answer D - Foundations of statistical analysis for the Health Sciences by Mendoza et. al p. 173
25. The highest score minus the lowest score equals the:
A. Percentile rank
B. Interquartile difference
C. Range
D. Standard deviation
Answer C - Foundations of statistical analysis for the Health Sciences by Mendoza et. al p. 183
26. The square root of variance:
A. Standard deviation
B. Interquartile range
C. Percentile
D. Kurtosis
Answer A - Foundations of statistical analysis for the Health Sciences by Mendoza et. al p. 188
27. This measure compares standard deviations between several investigations examining the
same variable:
A. Coefficient of variation
B. Fractional integer
C. Frequency distribution
D. Quartile deviation
Answer A - Foundations of statistical analysis for the Health Sciences by Mendoza et. al p. 189
28. All of the following are true of the normal distribution EXCEPT:
A. It is symmetrical about the mean
B. It is completely determined by 2 parameters: the mean and the standard deviation
C. It has different values for the mean, median, and the mode
D. It has long tapering tails that extend infinitely in either direction without touching the X
axis
Answer C Foundations of statistical analysis for the Health Sciences by Mendoza et. al pp.
208-209
29. The weight of Filipino babies at birth was measured and found to have a mean of 2500 grams
and a standard deviation of 300 grams. Which of the following statements is true?
A. 68.5% of the babies would have a weight ranging from 2200 to 2500 grams
B. 95% of the babies would have a weight ranging from 1900 to 3100 grams
C. 99.7% of Filipino babies would have a weight between 2200 and 2800 grams
D. All of the babies measured will have a weight between 2200 and 2500 grams
Answer B - Foundations of statistical analysis for the Health Sciences by Mendoza et. al p. 211

69

30. Which of the following measures central of tendency will have the highest value in a
distribution that is skewed to the left?
A. Average
B. Mean
C. Median
D. Mode
Answer D - Foundations of statistical analysis for the Health Sciences by Mendoza et. al p. 179
31.First step in a research process
a. Introduction
b. Research objective
c. Hypothesis
d. Conceptual definition
e. Research problem
Answer C- High-Yield Biostatistics by Lippincott Williams& Wilkins p.33
32.The frequency distribution of the sample mean that is generated by performing
repeated sampling and obtaining the mean from each sample is called
a. Sampling distribution
b. Percentage tabulation
c. Frequency tabulation
d. None of the above
Answer A- High-Yield Biostatistics by Lippincott Williams& Wilkins p.21
33.This refers to a statement about the value of a population parameter which is either rejected or
not rejected in hypothesis testing
a. Research question
b. Null hypothesis
c. Alternative hypothesis
d. Statistical decision
Answer B- High-Yield Biostatistics by Lippincott Williams& Wilkins p.33
34.The test statistic that is used to compare the proportions of three different groups is
a.
b.
c.
d.

z-test
t-test
Chi-square test
Linear regression

Answer C- High-Yield Biostatistics by Lippincott Williams& Wilkins p.46


35.The process of determining a range of values such that the parameter of interest is included in
this range is called
a. Interval estimation
b. Point estimation
c. Hypothesis testing
d. Critical region
Answer A- High-Yield Biostatistics by Lippincott Williams& Wilkins p.53
37.In hypothesis testing, this refers to the probability of obtaining the observed result or a more
extreme one under the null hypothesis
a. type I error
b. power
c. p-value
70

d. type II error
Answer C- High-Yield Biostatistics by Lippincott Williams& Wilkins p.37
38.In a study to compare the efficacy of two drugs, the statement The proportion of patients
that are cured by Drug A is the same as that by Drug B, is called a
a.
b.
c.
d.

Null hypothesis
Alternative hypothesis
Statistical decision
Research synthesis

Answer A- High-Yield Biostatistics by Lippincott Williams& Wilkins p.33


39.The first step in hypothesis testing is
a. State the level of significance
b. Specify the test statistic
c. State the alternative hypothesis
d. State the null hypothesis
Answer D- High-Yield Biostatistics by Lippincott Williams& Wilkins p.33
40.Which of the following statements is true about confidence intervals?
a.The higher the confidence level, the wider the interval estimate.
b.The higher the confidence level, the narrower the interval estimate.
c.The confidence level does not affect the width of the confidence
interval.
d.None of the above
Answer A- High-Yield Biostatistics by Lippincott Williams& Wilkins p.25

41.Incidence of a disease in a particular population over a specified time period:


a.Morbidity rate
C. Attack rate
b.Mortality rate
D. Crude rate
Answer A- High-Yield Biostatistics by Lippincott Williams& Wilkins p.72
42.The cumulative incidence of the disease when the duration of a disease is short and the
observation period covers an entire epidemic:
a. Case fatality rate
C. Mortality rate
b.Attack rate
D. Morbidity rate
Answer B- High-Yield Biostatistics by Lippincott Williams& Wilkins p.70
43..An individual who has contact with or who manifests the risk factor prior to becoming ill:
a.Exposed
C. Infected
b.Inflicted
D. Well
Answer A- High-Yield Biostatistics by Lippincott Williams& Wilkins p.72
44.The infant mortality rate of a country in a given year is 45.1 per year per 1000 live birth
means:
a.45.1 infants died before reaching their first birthday
b.45.1% of all deaths occurred in infants below one year
c.45.1 infants died before reaching their first birthday per 1000
live births
71

d.45.1 infants per 1000 of the population died


Answer A- High-Yield Biostatistics by Lippincott Williams& Wilkins p.72
45.An extraneous variable is called
a. Independent variable
b.Confounding variable
c.Dependent variable
d.Exposure variable
Answer B- High-Yield Biostatistics by Lippincott Williams& Wilkins p.60
46. . Performance of new born screening should be done :
A. 24 hours after birth but not more than one week
B. 24 hours after birth but not more than five days
C. 24 hours after birth but not more than three days
D. anytime after birth
E. none of the above
Ans. C- http://www.doh.gov.ph/health_laws.htm
47.The variable assumed to influence the problem
a.Independent variable
b.Confounding variable
c,Dependent variable
d.Continuous variable
Answer B- High-Yield Biostatistics by Lippincott Williams& Wilkins p.58
48. Newborn Screening Act of 2004 is also known as:
A. RA 9288
B. RA 9388
C. RA 9287
D. RA 9387
Ans. A- http://www.doh.gov.ph/health_laws.htm
49.. Penalties imposed in on individuals who violate the clean air act are the following except:
A. first offense one thousand pesos
B. second offense not less than two thousand pesos but not more than four thousand
C. third offense suspension of drivers license
D. fourth offense imprisonment of three months
Ans. D- http://www.doh.gov.ph/health_laws.htm
50.The assumption of the relation between two variables to be studied
a. Introduction
b.Research objective
c.Hypothesis
d.Conceptual definition
e. Research problem
Answer C- High-Yield Biostatistics by Lippincott Williams& Wilkins p.33
51. An Act which was design to promote corneal transplantation in the country:
A. Corneal Transplant ACT
B. Organ Transplantation ACT of 1998
C. Enacted on February 20,1995
D. Enacted on May 20,1995
Ans. C- http://www.doh.gov.ph/health_laws.htm
52. A study that describes an interesting or unusual condition in a group of patients
72

A. Case report
B. Ecological study
C. Cross-sectional study
D. Case control study
E. Case series
Answer E- High-Yield Biostatistics by Lippincott Williams& Wilkins p.64
53.Recall bias is a common problem in what type of study
a. Case report
b. Ecological study
c. Cross-sectional study
d. Case control study
e. Cohort study
Answer D- High-Yield Biostatistics by Lippincott Williams& Wilkins p.63
54. A study that establishes the association between risk factor and outcome variable
a.Case report
b.Ecological study
c.Cross-sectional study
d.Case control study
e.Cohort study
Answer E- High-Yield Biostatistics by Lippincott Williams& Wilkins p.62
55.In epidemiologic studies, the identification of causal factors is based on the experience of
a.An individual
b.A group of individuals
c. Experimentation
d. None of the above
Answer B- High-Yield Biostatistics by Lippincott Williams& Wilkins p.68

56.This refers to statistical procedures that allow valid generalization of results from a sample to
the population it is supposed to represent
a.Statistical Association
b.Causal Association
c.Descriptive statistics
d.Statistical inference
Answer d- High-Yield Biostatistics by Lippincott Williams& Wilkins p.1
57.When one sets of the maximum probability of making an error of rejecting the null hypothesis
when it is true, he is specifying the
a.Critical region
b.Statistical test
c.Level of Significance
d.Power
Answer C- High-Yield Biostatistics by Lippincott Williams& Wilkins p.34
58.In statistical inference, this term refers to the total collection of elements of interest
a.Population
b.Sample
c.Parameter
d.Statistic
73

Answer a Answer d- High-Yield Biostatistics by Lippincott Williams& Wilkins p.1- High-Yield


Biostatistics by Lippincott Williams& Wilkins p.1
59.This refers to the procedures in assessing evidence from a sample, whether or not the data
from the sample supports a hypothesis about a population
a.Statistical inference
b.Estimation
c.Hypothesis testing
d.Descriptive statistics
Answer c- High-Yield Biostatistics by Lippincott Williams& Wilkins p.33
60.In a sample of 500 Grade I children examined, 450 were found to have dental caries. The
proportion 90% computed by dividing 450 by 500 is an example of
a.Population
b.Sample
c.Parameter
d.Statistic
e. unskilled workers
Answer d- High-Yield Biostatistics by Lippincott Williams& Wilkins p.21
61.Double burden of disease means
a. While the patient has to suffer the physiologic effects of an illness, he also has to
contend with its psychological and economic impact
b. While infectious diseases of poverty still abound, degenerative illnesses are
on the rise.
c. The top causes of morbidity always comes in a pair of related illnesses
d. An infectious disease may be aggravated by a concomitant degenerative illness.
Ans. B- Annual Report of the Field Health Service Information System pp 113-122 . Sec.
Francisco Duque III
64.Which of the following does not describe the CURRENT state of the Philippine health
situation?
a.Double burden of disease
b.Continuing threat from emerging and resurgent diseases
c.Highly centralized health care system
d.Large variations in health status across population and socioeconomic groups
e.Health status generally improving but at a slower rate compared to neighboring
Asian countries
Ans. C- Annual Report of the Field Health Service Information System pp 113-122 . Sec.
Francisco Duque III
65.In relation to the preceding question, which agency takes the principal responsibility for
primary health care and service delivery at the community level as accorded by law?
a.Local Government Chief Executives (like the Mayor, Governors)
b.The Department of Health Secretary
c.The President of the Republic
d.The Doctors and midwives in the community
Ans. D- Annual Report of the Field Health Service Information System pp 113-122 . Sec.
Francisco Duque III
66.After infancy, this is the most common cause of disease and deaths for young children,
especially below 5 years of age
a.Immunizable Infectious disease
b.Degenerative diseases
74

c.Emerging diseases like Avian Flu


d.Malnutrition
Ans. D- Annual Report of the Field Health Service Information System pp 113-122 . Sec.
Francisco Duque III
67. RA 7883 is also known as:
A. Medical workers benefits and compensation act
B. Barangay Health Workers Incentives and Benefits Act of 1995
C. Barangay Health Workers Benefits Act of 2003
D. Voluntary Blood Donation Act of 2002
Ans. B- http://www.doh.gov.ph/health_laws.htm
68. Hospital Licensure Act, which requires that all hospitals be licensed is also known as:
A. RA 4226
B. RA 2426
C. RA 2264
D. RA 1235
Ans. A- http://www.doh.gov.ph/health_laws.htm
69. The National Blood Services Act was enacted into law in:
A. 1996
B. 1995
C. 2000
D. 1994
Ans. D- http://www.doh.gov.ph/health_laws.htm
70. An Act establishing Philhealth is also known as:
A. Philippine Health Insurance Act of 1995
B. National Health Insurance Act of 1995
C. National blood donation Act of 1995
D. Philippine Health Insurance and Benefits Act of 1995
Ans. D- http://www.doh.gov.ph/health_laws.htm
71. Traditional and Alternative medicine act of 1997 established this to promote traditional and
alternative medicine.
A. TAMATH
B. PATIHC
C. PITAHC
D. TAMHC
Ans. C- http://www.doh.gov.ph/health_laws.htm
72. TAMA was established in:
A. June 28, 1997
B. August 28,1996
C. September 30,1998
D. July 28, 1997
Ans. D- http://www.doh.gov.ph/health_laws.htm
73. Republic Act 8344 is an amendment to :
A. PB 107
B. PB 239
C. PB 701
D. PB 578
Ans. B- http://www.doh.gov.ph/health_laws.htm
73. On of the policies of this Act is to protect the consumer from deceptive, unfair and
unconscionable sales act and practices.
A.RA 7393
75

B. Consumer Act of the Philippines


C. Both
D. None of the Above
Ans. B- http://www.doh.gov.ph/health_laws.htm
74.The level of state involvement is lowest in which policy instrument?
a.Auction of property rights
b.Information and exhortation
c.Subsidy
d.Regulations
75. An Act regulating the packaging, use, sale and distribution of tobacco and other related
products.
A.RA 9211
B. PB 9211
C. tobacco protection act of 2003
D. Tobacco Distribution Act of 2002
Ans. A- http://www.doh.gov.ph/health_laws.htm
77. The Magna Carta of Public Health Workers, also known as RA 7305 was enacted on:
A. April 24,1993
B. March 26, 1992
C. April 24, 1994
D. May 1,1993
Ans. B- http://www.doh.gov.ph/health_laws.htm
78. To promote and encourage voluntary blood donation is one of the policies of the:
A. Blood Donation Act
B. Organ Donation Act
C. Blood Services Act
D. Organ Services Act
Ans. C- http://www.doh.gov.ph/health_laws.htm
79. An Act regulating the practice of pharmacy in the Philippines:
A. RA 5678
C. RA 5921
B. RA 5932
D. RA 5934
Ans. C- http://www.doh.gov.ph/health_laws.htm
80. RA 8344 is also known as:
A. National AIDS Prevention Act
B. Tobacco Distribution Act
C. Act Preventing Hospital Deposits
D. Clean Air Act
Ans. C - http://www.doh.gov.ph/health_laws.htm
81. Conducting an ocular view of all existing hospitals in the Philippines is one of the duties of
the :
A. DPWH
B. Department of Sanitation
C. DOH- Hospital Services
D. Hospital Licensure Board
Ans. C- http://www.doh.gov.ph/health_laws.htm
82. An act to promote, require and ensure the adequate supply, distribution, use and acceptance
of drugs and medicines identifies by their generic names.
A. Generics Act
B. Pharmacy Regulation Act
C. AIDS Act
D. RA 6665
76

Ans. A- http://www.doh.gov.ph/health_laws.htm
83. An Act prohibiting the sale, distribution and manufacture of counterfeit drugs:
A. RA 8302
B. RA 8023
C. RA 8203
D. RA 8032
Ans. A- http://www.doh.gov.ph/health_laws.htm
84. The Clean Air Act of 1999 was enacted on:
A. June 23, 1998
B. July 28,1999
C. July 27,1999
D. July 27,1998
Ans. D- http://www.doh.gov.ph/health_laws.htm
86. Included in the mandate of this act is the reporting of HIV/ AIDS positive individuals for the
purpose of monitoring AIDS cases in the Philippines:
A. HIV monitoring and control act of 1997
B. Philippine AIDS Prevention and Control Act of 1998
C. Philippine AIDS Control Act of 1998
D. AIDS ACT of 1997
Ans. B- http://www.doh.gov.ph/health_laws.htm
87.When was the Alma Ata declaration initially intended to be achieved?
a. Year 1990
b. Year 2000
c. Year 2015
d. Undetermined
Ans B -Health for all beyond 2000: the demise of the Alma-Ata Declaration and Primary Health
Care in developing countries; John Hall and Richard Taylor entitled "
88.When are the Millennium Development Goals (MDGs) initially intended to be achieved?
a.Year 1990
b.Year 2000
c.Year 2015
d.Undetermined
Ans. C-HTTP://www.un.org/millenniumgoals
89.Which of the following is not a direct health-related MDGs?
a.Reduce child mortality by two-thirds among children under five
b.Improve maternal health by reducing maternal mortality by three-quarters
c.Combat HIV/AIDS, Malaria, and other diseases
d.Achieve universal primary education
Ans. d-HTTP://www.un.org/millenniumgoals
90.Which of the following is not an MDG?
a.Achieve universal primary education
b.Promote gender equality and empower women
c.Combat HIV/AIDS, Malaria, and other diseases
d.Achieve Universal health promotion activities
Ans. D-HTTP://www.un.org/millenniumgoals
91.Which approach enumerated below emphasizes the need for community participation in
achieving betterment for health?
a.Primary Care Approach
b.Millennium Development Goals
c.Primary Health Care approach
77

d.Eradication of Extreme Poverty and Hunger


92.Which approach enumerated below emphasizes the need for investments in health to achieve
national development?
a.Primary Care Approach
b.Millennium Development Goals
c.Primary Health Care approach
d.Eradication of Extreme Poverty and Hunger
Ans. B-HTTP://www.un.org/millenniumgoals
93.Jeffrey Sachs, a Harvard-educated economist, authored the document commissioned by the
World Health Organization (WHO) that eventually established the global funds. The said funds
pooled monetary and various resources together at the global level and earmarked these into
Programs and activities targeted to eradicate debilitating diseases, especially in aid of developing
countries. What is the title of his document?
a.WHO Commission on Macroeconomics and Health
b.WHO Commission on Health-related Economics
c.Primary Health Care
d.The Commission on the Millennium Development Goals
Ans. A.-MACROECONOMICS AND HEALTH: INVESTING IN HEALTH FOR ECONOMIC
DEVELOPMENT by Jeffrey Sachs pp 1-17 Executive Summary
94.The MDGs stipulated reductions in maternal and child mortalities by a fraction of a
benchmark value. The MDGs are compared to the situation during which year that served as
benchmark?
a.1990
c.2015
b.2000
d.undetermined
Ans. A.-HTTP://www.un.org/millenniumgoals
95.Which of the following statements is false?
a.The document that served as basis for the MDGs showed evidences between the
links of disease and poverty.
b.The Alma Ata Declaration is obsolete since it should have been achieved in
the year 2000.
c.The Alma Ata Declaration uses the primary health care approach.
d.The Millennium Development Goals is conceptualized behind the idea that
better health means better productivity and vice-versa.
e.The MDGs push us to achieve better investments for health.
Ans B -Health for all beyond 2000: the demise of the Alma-Ata Declaration and Primary
Health Care in developing countries; John Hall and Richard Taylor entitled "
96.Which of the following is not an essential element of primary health care?
a.Immunization
b.Essential Medicines
c.Maternal Mortality
d.Water and Sanitation
Ans C -Health for all beyond 2000: the demise of the Alma-Ata Declaration and Primary
Health Care in developing countries; John Hall and Richard Taylor entitled "
97.Which of the following is not an essential element of primary health care?
a.Nutrition and supplemental feeding
b.Essential Medicines
c.Maternal and Child Health
d.Population Control and Management
Ans D -Health for all beyond 2000: the demise of the Alma-Ata Declaration and Primary
Health Care in developing countries; John Hall and Richard Taylor entitled "
78

98.Which of the following is not an essential element of primary health care?


a.Immunization
b.Endemic Diseases
c.Universal Primary Education
d.Curative Care
Ans C -Health for all beyond 2000: the demise of the Alma-Ata Declaration and Primary
Health Care in developing countries; John Hall and Richard Taylor entitled "
99.What year was the Alma Ata Declaration adopted by over 130 WHO Member-States,
including the Philippines?
a.1958
b.1968
c.1978
d.1988
e.1998

79

Ans C -Health for all beyond 2000: the demise of the Alma-Ata Declaration and Primary
Health Care in developing countries; John Hall and Richard Taylor entitled "
100.According to the WHO, health is defined as:
a..Not merely the absence of disease
b.The absence of infirmity
c.The complete physical, mental and social well-being of a person
d.All of the above
e.None of the above
Ans D-Health for all beyond 2000: the demise of the Alma-Ata Declaration and
Primary Health Care in developing countries; John Hall and Richard Taylor entitled "
1. In what case should a family case conference or meeting be of least help?

A. child admitted for LBM and vomiting of three days


duration
B. Two year old newly diagnosed to have thalassemia, a blood disorder
C. Fifty three year old working mother diagnosed with Stage III breast CA
D. Forty five year old father with TB who remains symtomatic on the 5 th
month of medication
2. The single most difficult time in the entire illness experience is
A. onset of illness

B. diagnosis of major debilitating or terminal illness


C. financial aspects of application of therapy
D. adjustment to permanency of outcome
3. Total evaluation of health care involving family relations, social and cultural
systems leading to better evaluation of health care
A. family life cycle

B. family systems theory


C. family set-up
D. family relationships/parent-child interaction
4. DOH program to control diarrhea is targeted for

A. under 5 years of age


B. children and pregnant women
C. lactating mothers
D. 2-14 years old
5. The following are covered by Philhealth except
A. legitimate married 19 year old daughter
B. thirty year old legally blind man
C. sixty five year old mother working at a sari-sari store but depends for
most needs on employed forty year old son
D. spouse who is not an NHIP member
6. In 1992 all administrative powers were transferred to political units and health
managers came under the management of non-health managers.
A. centralization

B. devolution
C. reorganization
D. restructuring

7. One of the guiding principles of RA 7875 is universality which refers to


A. adequately meeting the needs for personal health services at various
stages of a members life
B. provision to all citizens the mechanism of gaining financial
access to health services
C. provision of uniform basic benefits
D. balancing economical use of resources with quality of care
8. Membership in the NHIP shall take effect
A. three days after payment of premiums

B. upon enrollment and payment of premiums


C. at the time the member gets sick
D. one month after enrollment and payment of premiums
9. One of the following characterizes the essential drug concept
A. should be implemented by those in the government sector
B. one could select about 250 essential drugs for 90% of the
diseases in the country
C. does not allow patients to choose from a list of generic drugs
D. discrepancy between availability of drugs and needs of population
10. The following are categories of health services granted to the member or his
dependents for in-patient hospital care

A. room and board


B. fee for privately hired nurse
C. personal preventive services
D. food supplements
11. Ability of the body to produce antitoxin with diphtheria vaccine
A. agent characteristic
B. host characteristic
C. host-agent characteristic
D. agent-environment characteristic
13. Referral to home care program of a diagnosed cancer patient
A. primary prevention
B. secondary prevention
C. tertiary prevention
D. primordial prevention
14. The amount of chopped fresh leaves of lagundi for adults is
A. two teaspoons
B. two tablespoons
C. four tablespoons
D. six tablespoons
15. One of the following is used as diuretic and anti-urolithiasis agent
A. ulasimang bato
B. sambong
C. yerba Buena
D. guava
16. Ampalaya leaves is recommended for
A. insulin-dependent diabetes mellitus
B. non-insulin dependent diabetes mellitus
C. non-toxic goiter

D. toxic goiter
17. One of the following medicinal plants contains vinca alkaloids and may be
used as an anticancer agent
A. adelfa
B. aloe vera
C. rosas de baybayon
D. tawa-tawa
18. When garlic is used to lower cholesterol, it should not be
A. broiled
B. soaked in vinegar
C. fried
D. raw
19. Stability of vitamins under refrigeration

A. agent-environment interaction
B. host-environment interaction
C. human host-agent interaction
D. agent-host-environment interaction
20. Second line of defense against disease agents
A. secretions
B. respiratory cilia
C. mucous membranes and nails

D. lymph glands
21. Per single period of confinement a general practitioner will be paid by the
Philhealth
A.
B.
C.
D.

500 pesos/day
150 pesos/day
300 pesos/day
200 pesos/day

22. Strategies to control TB in the Philippines except


A.
B.
C.
D.

DOTS for all patients


Free medicines available for patients in public hospitals only
Active participation of private practitioners in NTP
NTP reaches out to physicians in private sector

23. Essential elements of primary health care except


A.
B.
C.
D.

MCH including EPI


Basic sanitation and safe water
Provision of food supply
Essential drugs available

24.One of the advantages of decentralizing the health system is


A. Greater representation of various political, religious, ethnic, and
tribal groups in development decision-making.
B. Bureaucratic and highly structured nature of the health system
facilitates more efficient delivery of services
C. Lesser participation of citizens in development planning and
management, hence, lesser conflict

D. Greater control of officials in the central office as regards to planning


and implementation of programs
25. According to Rule IX of the Magna Carta, a public health worker who is
regularly employed under permanent status shall have security of tenure
A. But he can be dismissed anytime from work if his boss finds him
unsatisfactory for the job
B. If he is unjustly dismissed from the service, he/she shall be
entitled to reinstatement without loss of seniority rights and
payment of back wages/salaries and other benefits
C. And can never be terminated from his job
D. But is not entitled to benefits and back wages if unjustly dismissed
from service
26. Family type that includes stepparents and stepchildren brought about by
annulment, separation and remarriage
A.
B.
C.
D.

nuclear
extended
blended
communal

27. True of systems concepts of family by Minuchin except


A. adapting to change is hallmark of healthy functioning
B. symptoms of our member have effect within the family
C. repeating interaction patterns regulate behavior of members
D. usually there are victims and victimizers in the family
28. Family structure where relationship exists between at least three people and
two collude against the other
A.
B.
C.
D.

coalition
alliance
role selection
boundaries

29. Enmeshment as family process occurs in the following


A.
B.
C.
D.

mother-child subsystem excludes father in parenting


third person comes into a two-pair system to diffuse conflict
members are overly reactive to stress on one member
parents have privacy from children

30. True of family genogram construction except


A.
B.
C.
D.

consist of at least three generations


firstborn is farthest to the right with sibling following to the left
horizontal lines connects children with spouses
children connected to parents with vertical lines

31. In APGAR, this measures the satisfaction attained in solving problems by the
communicating
A.
B.
C.
D.

Affection
Resolve
Adaptation
Partnership

32. A mother who insists to accompany her son enrolling in medicine proper
during the routine physical exam in school
A. Triangulation
B. Coalition
C. Disengagement
D. Enmeshment

33. A daughter sides with her mother regarding their problem with the father.
They accuse the father having another woman
A.
B.
C.
D.

coalition
disengagement
triangulation
enmeshment

34. A family with three children ages 9, 10, and 15 living with their
grandparents in the city have adopted a 2-year old boy. The familys APGAR
is 3. This familys function is
A.
B.
C.
D.

normal
mildly dysfunctional
moderately dysfunctional
severely dysfunctional

35. Atherosclerotic changes in coronary vessels


A. Stage of Susceptibility
B. Stage of Presymptomatic disease
C. Stage of Clinical disease
D. Stage of Disability
36. Measles followed by the development of subacute sclerosing panencephalitis
A. Stage of Clinical disease
B. Stage of Susceptibility
C. Stage of Presymptomatic disease
D. Stage of Disability
37. Secondary prevention except
A. specific protection
B. disability limitation
C. prompt treatment
D. early diagnosis
38. Health promotion except
A. use of immunizations
B. genetics
C. personality development
D. periodic selective examination
39. Levels of preventive measures affecting reaction of the host to the
stimulus except
A.
B.
C.
D.

specific protection
rehabilitation
disability limitation
early diagnosis and prompt treatment

40. Tertiary prevention except

A.
B.
C.
D.

sheltered workshop
half-way homes
occupational therapy
improvement in working conditions in dusty industries

41. Early diagnosis and prompt treatment in cancer include the following except
A. Recruitment and training of specialists
B. Self-examination of breast
C. Cancer detection centers
D. Selective examination
42. Ability to stimulate the host to produce agglutinins, opsonins, antitoxins, etc.
A.
Pathogenicity
B.
Virulence
C.
Antigenicity
D.
Infectivity
43.Measure of ability of an agent when lodged in the body to set up local or
general tissue reaction
A. Pathogenicity
B. Virulence
C. Infectivity
D. incubation period
44. The most relevant part of the death certificate because it is the one counted
in the statistics since it is the basic condition which began the train of events
which led to death
A.
immediate cause of death
B.
intervening cause of death
C.
intermedial cause of death
D.
underlying antecedent cause
45. Which of the following is true regarding physiologic adjustments in
temperature?
A. The percent of the body surface wetted with sweat decreases with the
need to lose heat until maximum sweating occurs.
B. The sweat, which runs off, the body is of great value in removing heat
from the body, since heat is lost only when sweat is evaporated.
C. If the temperature and humidity are both high, the body may be
unable to lose sufficient heat.
D. Men do not voluntarily replace the water lost by sweating; the
sensation thirst is quite adequate.
46. Relation of high temperature and humidity to physical work
A. Muscular work contributes little to the total body heat load than
environmental conditions
B. Rectal temperature is most closely related to the rate of physical
work
C. The capacity of a man to work drops off at high temperature because
of lack of oxygen supply.
D. A marked increase in lactic acid in the blood limits the capacity of man
to work at high temperature.
47. If the body is immersed in cold water, rather than cold air
A. body temperature falls gradually
B. air conducts heat faster than water
C. air has a higher specific heat value

D. entire body surface loses more heat to the water


48. This is the 2nd most abundant greenhouse gas:
A. nitrous oxide
B. CFCs
C. Carbon dioxide
D. Methane
49. This is not necessary in the formation of Ozone
A. hydrocarbons
B. SO2
C. Nitrous oxide
D. Sunlight
50. The process of removing or destroying all microbial life of an object
A. antisepsis
B. sterilization
C. disinfection
D. antimicrobial
51. Exert their action by injuring plasma membranes
A. phenols
B. biguanides
C. halogens
D. alcohols
52. True on halogens as disinfectants except
A. Iodine may combine with certain amino acids to inactivate enzymes
B. Germicidal action of chlorine is based on the formation of
hyperchlorous acid when chlorine is added to water
C. Iodine is available as a tincture (combined with an organic
molecule).
D. Chlorine is used as a disinfectant in gaseous form
53. The following statements are correct regarding surface-active agents except
A. Acid anionic detergents are used to clean dairy equipment
B. Soaps assists in the removal of microorganisms through scrubbing
C. Detergents decrease the surface tension among molecules of a liquid
D. Soaps have a wide range of germicidal action
54. True concerning radiation except:
A. Microwave can kill microbes indirectly as materials get hot.
B. The effects of radiation depend on wavelength and duration
C. Ultraviolet radiation has a high degree of penetration and exerts
primary by ionizing water
D. X-rays exert its effect by forming highly reactive hydroxyl radicals
55. The seven target diseases are:
A. measles, polio, pneumonia, tetanus, chicken pox, TB
B. diphtheria, mumps, polio, Tb, tetanus, measles
C. TB, polio, measles, diphtheria, pertussis, tetanus, hep B
D. Tetanus, pertussis, tonsillitis, measles, tetanus, TB
56. Extremely low frequency electromagnetic fields can cause brain tumor and
this is emitted by
A. television sets

B. laser printers
C. Xerox machines
D. Microwave oven
57. A 20-year-old female in respiratory distress was brought to the emergency
room for intentional ingestion of a poisonous substance. Of the following,
which is the most appropriate thing to do first.
A.
B.
C.
D.

Get a detailed history


Maintain vital signs
Give antidote
Induce vomiting

58. Contamination of food by cockroaches is as example of :


A. vehicle-borne transmission
B. air-borne transmission
C. mechanical transmission
D. biological transmission
59. Gas responsible for explosion in garbage dumps:
A. carbonic acid gas
B. carbon monoxide
C. methane
D. nitrogen
60. Usual speech frequency is:
A. 0 db
B. 120 db
C. 500 to 2000 Hz
D. 20 to 20,000 Hz
61. Most likely to occur during cases of bomb explosions:
A. conductive hearing loss
B. noise induced deafness
C. sensorineural hearing loss
D. temporary threshold shift
62. In storing vaccines, the following should be remembered
A. DPT vaccine can be stored in the freezer and thawed
B. The diluent should be in the area where the solvent is
C. Never keep vaccine in the door of the refrigerator
D. If the vaccine stored above or below safe temperature it will not lose its
potency if it is for a short period of time only.
63. In transporting vaccine using a vaccine carrier this vaccine should be placed
in contact with frozen ice packs
A. BCG
B. OPV
C. DPT
D. TT
64.This vaccine is most easily damaged by sunlight
A. polio vaccine
B. BCG
C. DPT
D. Tetanus toxoid

65. A fully immunized child has finished


A. BCG, 2 DPT, 2 OPV
B. BCG, 3 DPT, 3 OPV
C. BCG, 3DPT, 3 OPV, Measles
D. 3 DPT, 3 OPV, Measles
66. A 21 year old woman is pregnant with her first child, she had had received 2
doses of DPT when she was a child. How many doses does she need so that
all infants born to her will be protected?
A. 2
B. 3
C. 4
D. 5
67. Normal reaction to BCG vaccination are the following except
A. small and tender swelling about 10mm across after 2 weeks
B. swelling became small abscess when vaccinated after 2-3 weeks
C. formation of scar after about 10 weeks
D. acute inflammation reaction appearing 4-2 days of vaccination
68. The separation of infected persons for the period of communicability to
prevent transmission of infectious agent to those who are susceptible is
A. quarantine
B. surveillance
C. isolation
D. herd immunity
69. The prevention transmission of highly contagious or virulent infection that
may be spread by both air and contact is
A. strict isolation
B. contact isolation
C. respiratory isolation
D. drainage precaution
70. These diseases are internationally quarantinable diseases and thus are
labeled as Diseases Under Surveillance by WHO.
A.
B.
C.
D.

plague, cholera, smallpox


plague, typhoid fever, cholera
leprosy, smallpox, plague
malaria, leprosy, smallpox

72. When shellfish is harvested from a contaminated area, it should purge before
consumption for approximately:
A. 1 day
B. 1 week
C. 2 weeks
D. 2 days
73. One of the neurotoxins found in coal:
A. cadmium
B. chromium
C. manganese
D. arsenic

74. The most important test for potability of water.


A. taste and odor
B. chemicals
C. algae, microorganism
D. strep coliforms
75. Examples of water-borne diseases EXCEPT :
A. amoebiasis
B. minimata disease
C. poliomyelitis
D. schistosomiasis
76. Man is the definitive host in:
A. dengue fever
B. filiariasis
C. malaria
D. Rocky mountain spotted fever
77. Rats

and other rodents are the reservoir and source of several diseases of
man except
A.
B.
C.
D.

schistosomiasis
salmonellosis
dengue hemorrhagic fever
plague

78. Which should be first priority in disaster relief?


A. Safety of food supply
B. Vector control
C. Excreta disposal
D. Promotion of personal hygiene
79. Early effects of ionizing radiation includes the following except
A. anorexia, nausea, vomiting
B. marrow syndrome
C. sterility
D. increase childhood cancer
80. The following are environmental sources of occupational stress except
A. excessive noise
B. attitudinal problems
C. hot/cold fever
D. posture
81. The bacterial limits for pasteurized milk and milk products is:
A. 100, 000/ml
B. 10,000/ml
C. 20,000/ml
D. 1gm/ml
82. A selective, partial limitation of freedom of movement of contacts, on the
basis of known or presumed differences in susceptibility and related to danger
of disease transmission.
A.
B.
C.
D.

absolute quarantine
modified quarantine
isolation
surveillance

84. What type of sampling technique is used for Man-in-the-street surveys and
interviews among senior citizens who drop in at a health booth in SM City?

A. simple random
B. systematic
C. haphazard
D. cluster
85. What is the ability of a study to demonstrate an association if one exists?
A. alpha
B. P value
C. power
D. Type II error
86. In contrast to observational studies, experimental research:
A. attempts to describe prevailing trends occurring in a population
B. empirically tests a cause-effect relationship among variables
C. requires two or more population groups in a study
D. identifies risk factors accompanying the independent variable
87. Any of the following may be included as an exposure in health research
except:
A. potential risk factor
B. The outcome being studied
C. The treatment or therapy
D. A health problem
88. The following are impact programs of DOH except
A. soil-transmitted helminthes disease control
B. rabies control

C. malaria control
D. environmental health services
89. Type of outbreak where exposure is brief and all cases develop within one
incubation period of the disease.
A. common source outbreak with intermittent exposure
B. common source outbreak with continuous exposure
C. point source outbreak
D. propagated outbreak
90. A distribution that is negatively skewed hasA. higher frequency on the right
B. higher frequency on the left
C. a flatter distribution than the normal distribution
D. a more peaked distribution than the normal distribution
91. Which has a wider interval?
A. 80% confidence interval
B. 90% confidence interval
C. 95% confidence interval
D. 99% confidence interval
92. Aims to reveal the existing gaps in knowledge regarding the problem being studied

A.
B.
C.
D.

significance of the study


research question
review of literature
conceptual framework

93. A nominal scale:


A. is made up of labeled or named categories with no implied order
B. is a ranking scale
C. has an ordering of values with one better than the other
D. has equal distances between values
94. In a stratified sampling:
A. k is determined by dividing the number of items in the sampling frame
by the desired sample size
B. the population is first divided into relevant subgroups and a
random sample is selected from each
C. the probability that a subject may be selected is unknown
D. the assignment of subjects to treatments is done by using random
methods
95. Which is used to test for differences among proportions?
A. paired t-test
B. B. 2-sample t-test
C. ANOVA
D. D. Chi-square test
96. An investigator conducts a historical cohort study to explore the relationship
between perimenopausal exogenous estrogen use and the risk of coronary
heart disease (CHD). A total of 5000 exposed and 5000 unexposed women
are enrolled and followed for 15 years for the development of myocardial
infarction (MI). A total of 200 estrogen users and 300 nonusers had MIs. What
is the relative risk for MI?
A.
B.
C.
D.

0.33
0.50
0.67
1.5

97. A study was undertaken to compare results of the surgical treatment of


duodenal ulcer. A total of 1,358 patients who met the study criteria were
randomly assigned to one of four surgical procedures. The purpose of the
randomization was to A. ensure that the double-blind aspect of the study was maintained
B. obtain the unbiased distribution of good- and poor-risk patients in
all treatment groups
C. achieve the same number of patients on each operation
D. guarantee that the study group was a representative sample of the
general population
98. A newly developed test for diabetes produced positive results in 138 of 150
known diabetics and in 24 of 150 persons known not to have diabetes.
What is the sensitivity of the test?
A. 84%
B. 85.2%
C. 88%
D. 92%

99. Treatment of a patient with gonorrhoea constitute:


A. primary prevention for the patient, secondary prevention for potential
contact
B. secondary prevention for patient, primary prevention for potential
contact
C. secondary prevention for patient, tertiary prevention for contact
D. tertiary prevention for contact, primary prevention for patient
100. An indirect health impact of climate change:
A. heat wave in Europe in 2003 causing more than 20,000 deaths
B. respiratory illness due to an increase in ozone
C. altered concentration of fungal spores causing respiratory problems
D. El Nio events causing dengue outbreaks

Vous aimerez peut-être aussi